Section 4.1 - Environment

अब Quizwiz के साथ अपने होमवर्क और परीक्षाओं को एस करें!

Describe the pathophysiology of heat exhaustion

"Heat exhaustion is characterized by the inability to maintain adequate cardiac output due to strenuous physical exercise and environmental heat stress. The body's ability to combat heat stress is overwhelmed - evaporation, conduction, convection, radiation - through vigorous activity, fitness level deficits, poor acclimatization to heat, heavy clothing/equipment, and compromised physiologic response (eg, degree of tachycardia). This leads to Uncompensated heat stress (UCHS): when cooling capacity is exceeded and the athlete cannot maintain a steady temperature. Continued exertion in the setting of UCHS increases heat retention, causing a progressive rise in core body temperature and increasing the risk for severe heat illness. The clinical criteria for heat exhaustion generally include the following: - Athlete has obvious difficulty continuing with exercise - Core body temperature is usually 101 to 104ºF (38.3 to 40.0ºC) at the time of collapse - No significant dysfunction of the central nervous system (eg, seizure, altered consciousness, persistent delirium) is present If any central nervous system dysfunction develops (eg, mild confusion), it is mild and resolves quickly with rest and cooling. At risk medications: • Anticholinergic agents • Antiepileptic agents • Antihistamines • Decongestants • Phenothiazines • Tricyclic antidepressants • Amphetamines • Ergogenic stimulants (eg, ephedrine, dimethylamylamine) • Lithium • Diuretics • Beta blockers • Ethanol - Uptodate

Describe the pathophysiology of heat stroke

"Large environmental heat load that can't be dissipated." As we discussed earlier there are two main forms of heat stroke: Classic form: - These elderly patients (usually >70 yrs) have a chronic illness that impairs their body's thermoregulation (they can't vasodilate, lose heat via sweating or increase their cardiac output); they are unable to get out of a hot environment/cool themselves/remain hydrated - Some young patients with psychiatric illness (clozapine use chronically) or drug addiction (ETOH and cocaine) are at risk for heat stroke. Exertional form - Occurs in otherwise healthy, usually young patients who exercise intensely in high temperatures and humidity. Remember that >75% humidity = heat can't be efficiently lost via evaporation. - Most of these patients have some risk factors for exertional heat illness Temperature elevation is accompanied by an increase in oxygen consumption and metabolic rate, resulting in hyperpnea and tachycardia. Above 42ºC (108ºF), oxidative phosphorylation becomes uncoupled, and a variety of enzymes cease to function. A cytokine-mediated systemic inflammatory response develops, and production of heat-shock proteins is increased. Blood is shunted from the splanchnic circulation to the skin and muscles, resulting in gastrointestinal ischemia and increased permeability of the intestinal mucosa. Hepatocytes, vascular endothelium, and neural tissue are most sensitive to increased core temperatures, but all organs may ultimately be involved. In severe cases, patients develop multi-organ system failure and disseminated intravascular coagulation (DIC)" - From UptoDate Risk factors for poor outcomes in heat stroke: - Elderly - Hypotension - Coagulopathy / DIC - Need for intubation

What is nitrogen narcosis?

- "Rapture of the deep" - Intoxicating effects of increased tissue nitrogen concentration - Signs and Symptoms: Euphoria, false well-being, confusion, poor judgement/skill, disorientation, laughter, diminished motor control, paresthesias/numbness in lips/gums/legs. - Typically occurs at depths below 100 ft when compressed air is used - Symptoms usually resolve with ascent, but can result in drowning, or dive emergencies because of poor motor control - A similar toxicity can occur with elevated partial pressures of oxygen for a long period of time; but this only occurs with deep dives without complex hypoxic gas mixtures.

What three mechanisms cause worsening bleeding in trauma in hypothermia?

- A physiologic hypercoagulable state occurs in the hypothermic patient. This can lead to DIC, leading to catastrophic bleeding - Diminished enzymatic activity and clotting factor activity is caused by diminished core body temperature. Remember, however, coagulation testing will be unreliable, as kinetic tests are performed at 37 degrees Celsius. Will be deceptively normal - Thrombocytopenias due to splenic sequestration also increase the risk for bleeding - Diminished activity of thromboxane A2 increases the risk for bleeding, particularly in trauma

List five causes of dizziness associated with diving.

- AGE - DCS type 2 (neurologic) - DCS type 1 (inner ear DCS) - Inner ear barotrauma (ear squeeze) - Anything else: i. Oxygen toxicity ii. Nitrogen narcosis iii. Hypoglycemia So, ask when the dizziness started! IF you can't figure it out, call hyperbarics and consider diving them!

Describe the basic pathophysiology of decompression sickness

- According to Boyle's and Henry's laws - the gases from a diver's breathing mixture are dissolved into the body i. Also, the longer a diver is at depth, the greater the amount of gas will dissolve into the tissues - With ASCENT most of the dissolved gas comes out of solution → if this is rapid the body can't accommodate all the bubbles leading to DCS; ****think of it like opening a bottle of carbonated beverage***** i. If the bottle is shaken and opened rapidly, it foams over and causes a mess (DCS), but if it's opened slowly then the pressure release is controlled (and the lung is able to clear the excess gas through its vascular bed). - DCS = decompression sickness i. A spectrum of clinical illnesses - due to the formation of small bubbles of nitrogen gas in the blood and tissues on ASCENT! ii. LOCATION, location, location of the bubbles - determines the type of symptoms that arise - Fetal circulation anatomy explains why pregnant women should not dive. - Multiple small bubbles normally occur on ascent, but if they become persistent, large or too numerous for the lungs to filter → inflammatory cascades ensue, cytokines, thrombosis, ischemia, obstruction etc. can occur. i. These bubbles can cause ischemia and hypoxia if large ***Nitrogen is highly fat soluble - so it can very easily absorb into the white matter of the CNS → leading to huge problems when the pressure suddenly drops.***

What are the indications for re-compression (hyperbaric treatment)? How does it work?

- Air gas embolism - Decompression sickness types 1 and/or 2 - Carbon monoxide poisoning (contaminated air) Reduction of gas bubble size — The use of hyperbaric oxygen therapy for decompression illness is based upon Boyle's Law, since the volume of nitrogen bubbles is inversely related to the pressure exerted upon it. At 3.0 atm, bubble volume decreases by approximately two thirds. Further bubble dissolution is accomplished by the replacement of inert nitrogen within the bubbles with oxygen, which is then rapidly metabolized by tissues. - uptodate

What are the types of electrical injury? (based on type of current)

- Alternating current injury: Including conducted energy weapon discharge - Direct current injury - Lightning injury

Differentiate between AC and DC current injuries

- Electrical sources create current that flows in one direction (direct current, DC) or alternates direction cyclically at varying frequencies (alternating current, AC). - The few systems that use DC include batteries, automobile electronics, and railroad tracks. Exposure to DC most frequently causes a single, strong, muscular contraction. This may throw the subject back from the source in a way that limits duration of exposure but can result in other injuries. Think blast/blunt trauma. - AC is more commonly used (eg, household currents) because it conveniently allows for an increase or decrease of power at transformers. - It is more dangerous than DC of similar voltage because amperage above the so-called let-go current will cause muscular tetanic contractions. - Because the flexor muscles of the upper extremities are stronger than extensor muscles, these contractions pull the victim closer to the source and result in prolonged exposure.

Describe the pathophysiology of frostbite (the freezing injury cascade).

- Frostbite only occurs when the tissue gets below 0ºC. (Usually more likely -4º to -10º C) - The tissues become injured due to ice crystal formation, microvascular thrombosis and stasis. - Here we walk about the "Freezing Injury Cascade" Pre-freeze stage: ii. Temps below 10 deg iii. CUTANEOUS SENSATION LOST iv. Microvascular changes (see below) Freeze thaw phase i. Ice crystals form outside the cell ii. Then inside the cell iii. Cells die iv. Blood flow stops v. As the tissue becomes thawed the next stage starts Vascular stasis and ischemia stage i. Coagulation in microcirculation ii. Damage tissue releases toxic mediators iii. Tissues become ischemic as the coagulation system is activated i. Tissue edema for 48-72 hrs as tissue is thawed ii. Necrosis appears as the edema resolves iii. Dry gangrene appears Advanced imaging may help delineate which tissue is viable before July rolls around.... (referencing the frostbite in January amputate in July adage) See Box 131.1: Freezing Injury Cascade *Extremely rapid cooling produces more initial intracellular than extracellular ice crystallization

List 6 admission criteria for electrical/lightning injuries

- History of high voltage injury or lightning strike - Altered mental status / neuro deficits - Respiratory weakness / paralysis - Cardiac injury / contusion / dysrhythmia / ischemia - Hypotension - Third spacing - Extensive thermal burns or circumferential burns - Rhabdomyolysis - Compartment syndrome - Blunt traumatic injuries - Extremity thrombosis or vasospasm - Deep pediatric oral burns (for hydration) - Pregnant patients

List 6 complications of high-voltage injuries

- Neuro-psychiatric sequelae - Blunt trauma (anywhere!) - Cardio-pulmonary arrest - Thermal burns (anywhere!) - Tetanic muscle contractions (leading to fractures, compartment syndrome) - Rhabdomyolysis

What are late complications of drowning?

- Secondary neurologic injury - ischemia, cerebral edema, seizures - Hypovolemia - due to cold diuresis - Rhabdomyolysis - Renal impairment - Aspiration pneumonitis - Respiratory infection - Hypoxic cardiomyopathy - PTSD

List 4 systemic diseases for whom high-altitude travel is contraindicated

- Sickle cell disease (with hx of crises) - Severe, symptomatic CAD (or other severe CHF - low EF, high grade ventricular ectopy) - Severe COPD - Symptomatic pulmonary hypertension - Uncompensated CHF

Describe 5 management considerations other than rewarming in hypothermia

-Safety of you, the extrication team or other victims that may take precedence - Identify and treat other potential life threats to the patient - ABC's i. Intubate ii. Supplemental 02 iii. Stop bleeding / volume resuscitate - Treat other cold-induced injuries (frostbite): Rewarm, analgesia, tetanus prophylaxis - Treat other wounds - Consider whether antibiotics are indicated: i. Young children and the elderly should be given prophylactic antibiotics ii. Routine empirical antibiotics do not appear warranted in hypothermic, non-older adults. Antibiotics should be administered if the clinical picture is consistent with septic shock or if there is failure to rewarm. Consider empiric IV hydrocortisone (in addition to antibiotics) in patients who fail to rewarm - Contact patient's family / friends and get collateral information

List risk factors for drowning

-Toddlers -Elderly (>75) (hurricane katrina deaths) - Boys - American Indian and Alaska Native - Seizure d/o - Autism or developmental disorders - Prolonged QT syndrome (immersion syndrome) or other cardiac mutations - Hypothermia - Hyperventilation before a shallow dive - Concomitant trauma, stroke, or myocardial infarction - No adult supervision - Risk taking behaviour - Summer months, on weekends, water sports - ETOH intake or drug intake - Inexperienced swimmers and very experience swimmers

Describe the modifications in field triage of multiple victims following a lightning strike.

.... the initial arrhythmia is asystole. At some point, the intrinsic pacemaker activity of the heart brings about a resumption of cardiac activity. However, if the respiratory center has not been reactivated, hypoxia follows, and the cardiac rhythm will deteriorate into ventricular fibrillation.. This may explain reports of successful resuscitation and full recovery of lightning strike victims after being apneic and pulseless for 15 minutes and following resuscitations lasting up to 8 hours. This observation has led to the practice of treating the apparent dead first at the scene of a multiple-victim lightning strike because early resuscitative efforts may prevent death.

Describe the management of a drowning patient with respiratory distress

...if you're on scene proceed with wilderness medicine principles* then initiate basic life support (get out of the water, start CPR when on a solid surface, only immobilize C-spines if: fall from a height, diving into shallow water, signs of trauma, motorized vehicle collision) - Note that the priorities of CPR in the drowning victim differ from those in the typical adult cardiac arrest patient, which emphasize immediate uninterrupted chest compressions. If the patient does not respond to the delivery of two rescue breaths that make the chest rise, the rescuer should immediately begin performing high-quality chest compressions. CPR, including the application of a automated external defibrillator, is then performed according to standard guidelines. - Uptodate - Check for that pulse in hypothermic patients for at least 1 minute! Assuming you're in the hospital: 1. If you have time - brief your team on what you know a. Expected injuries b. Expected management priorities c. What to do if the patient deteriorates 2. MOVIE! a. Goal Sp02 > 94% 3. ABC vs. H+P patient a. Resuscitate → consider whether this is a medical or a trauma resus! i. Circulation 1. Start CPR! 2. Core temp a. Rewarming only up to 34°C followed by a 24-hour mild hypothermic treatment before normothermia is reached may be advantageous because of decreased pulmonary reperfusion injury and reduced secondary brain injury. Emerging resuscitation literature indicates an emerging role for therapeutic hypothermia in drowning victims. ii. Airway 1. Know that a lot of water can be swallowed, so consider decompressing with an NG prior to excessive bagging! a. Aspiration of gastric contents greatly compounds the degree of pulmonary injury and increases the risk of ARDS iii. Breathing 1. Look for signs of pulmonary injury - hypoxia, cyanosis, resp. Distress a. Rhonchi, wheezes, rales → things may be getting worse! iv. Disability 1. Cerebral focused resuscitation a. Avoid the killers! 2. Injury of the cervical spine is not common in patients with submersion injuries, but precautions should be taken if there is: a. concerning history (eg, dive into shallow water) or signs of injury. b. If you have time: i. Get a full SAMPLE history and event details ii. Assess - Consider what precipitated this drowning: a. drug or ethanol intoxication, cardiac arrest, hypoglycemia, seizure, and attempted suicide or homicide, NAT, C-spine injuries iii. Workup iv. Admit or treat and street! 1. Asymptomatic patients should probably be observed for 6-8 hours to watch for delayed pulmonary injury (follow vital signs, follow clinical exam and consider repeat CXR prior to d/c) Note what is NOT above (i.e. no evidence for it!) - Steroids - Diuresis - Empirical antibiotics (unless in grossly contaminated water or signs of infection) Additional workup: - ABG - Labs with renal function - ECG (QTC!) - CXR (with repeat testing in a few hours if signs of pulmonary involvement on exam or vital signs) - Consider getting that CT head (and maybe Cspine) if the patient is stable to look for any pre-existing bleed, trauma, or cerebral edema (this may help our ICU colleagues prognosticate in 24 hrs) Victims with CNS injury may present with symptoms ranging from mild lethargy to coma with fixed and dilated pupils. CNS injury results from the initial hypoxic or ischemic insult and from the cascade of reperfusion injury that follows reestablishment of cerebral blood flow after an arrest. The release of inflammatory mediators and the generation of oxygen free radicals in the post resuscitation period contribute to cytotoxic cerebral edema, compromise of the blood-brain barrier, and increased intracranial pressure. Cardiac dysrhythmias may incite drowning or develop as its consequence. Hypoxemia, acidosis, and, potentially, hypothermia are the primary factors responsible for dysrhythmias ranging from ventricular tachycardia and fibrillation to bradycardia asystole. Electrolyte disturbances are rarely significant enough to be dysrhythmogenic. Other clinical sequelae of drowning may include acute renal impairment, which is present on admission in approximately 50% of patients as the result of lactic acidosis; prolonged hypoperfusion; and, in some instances, rhabdomyolysis. Coagulopathy as a consequence of associated hypothermia or disseminated intravascular coagulation (DIC) may also occur.

Describe the following laws:

...let's talk about some laws of physics - defining the behaviour of liquids and gases...This first one is easy, think of it like a tube a toothpaste. Since most of our body is made of water...... a. Pascal's Law : ΔP = pg(Δh) i. A pressure applied to any part of a liquid is transmitted equally throughout. We care because....as the pressure increases in a confined space (e.g. the inner and middle ear) it will cause barotrauma to the adjacent structures - inner/middle ear structures. These adjacent pressure changes affect the volume of air filled spaces in the body (lungs, bowel, sinuses, middle ear). These fragile structures (just like any other hollow tube (tire, balloon)) don't do well when they are overfilled! b. Boyle's Law : P1V1 = P2V2 (at a constant temp) i. The pressure and volume of a gas are inversely proportional to each other. ...the volume of a gas (air) will decrease as the pressure (due to depth) increases. **this also applies to changes that occur in gas supplies** We care because as dives deeper (more pressure), the gas bubble volume will decrease. The greatest change happens in the first 33 ft (100% → 50% volume change) ii. Low pressure = high volume; high pressure = low volume. c. Charles' Law: V1/T1 = V2/T2 i. Addresses the effect of temperature on gases: "at a constant pressure, the volume of a gas is directly proportional to the change in absolute temperature. ii. Therefore, with heat the volume of a gas will increase. d. General Gas Law: P1 x V1/T1 = P2 x V2/T2 i. This law combines charles' boyle's gas laws. It relates pressure, volume and temperature in one equation when they aren't constant. e. Dalton's Law: Ptotal = P1 + P2 + P3 etc... i. The total pressure exerted by a mixture of gases is equal to the sum of their individual partial pressures. We care because nitrogen under pressure acts as if no other gases are present. f. Henry's Law: ep=ekc i. ...the amount of any gas that dissolves in a liquid at a given temperature is directly proportional to the partial pressure of that gas. ii. We care because more nitrogen is taken into solution (your blood) at higher pressures, than comes out of solution at lower pressures. iii. At higher pressures, the concentration of each component of air in blood and tissues increases until a steady state is reached. If you're only going to remember two; I'd suggest these two: - Boyle's law states that at a constant temperature, the volume of a gas varies inversely with the pressure to which it is subjected. This law helps to explain the principles behind diving-related barotrauma and air embolism. - Henry's law states that at a constant temperature, the amount of a gas that is dissolved in a liquid is directly proportional to the partial pressure of that gas. This law provides the explanation for decompression sickness and nitrogen narcosis.

Describe the risk of high altitude in the following patient groups:

1. Coronary artery disease a. In theory, (no studies on this) people with diseased myocardium should be advised to avoid high altitude because of decreased environmental oxygen availability. b. Due to the hypoxic response travellers will have increased sympathetic activity, increase in their cardiac work demands and increased myocardial oxygen consumption c. increase angina symptoms and dysrhythmias. Although both cardiac rhythm abnormalities and ST segment and T wave electrocardiographic changes are reported, none of these changes are associated with clinical evidence of myocardial ischemia. Limited data suggest no increased risk for sudden cardiac death or myocardial infarction at altitudes up to 8000 feet. When individuals with stable angina are exercised, there is conflicting evidence for the probability of inducing malignant dysrhythmias. Travellers with heart disease who ascend to moderate altitudes do not appear to have an increased incidence of AMS. d. Travellers with mild stable CAD should be advised to ascend gradually, to limit activity especially in the first few days at elevation, and to continue anti-anginal and antihypertensive medications. Individuals who have more severe, symptomatic coronary disease or those in a high-risk group (low ejection fraction, abnormal stress test results, and high-grade ventricular ectopy) should avoid travel to high altitudes. Ascent to moderate elevations can be suggested on an individual basis with the previously mentioned precautions. Individuals with heart failure who travel to altitude may require increased use of diuretics to promote diuresis and acclimatization. Acetazolamide prophylaxis may be useful to speed acclimatization and to prevent AMS and its accompanying fluid retention. 2. Pulmonary hypertension a. At increased risk for HAPE - they should be advised AGAINST travel to higher elevations b. If travel necessary use oxygen, nifedipine, PDE5 inhibitors, steroids 3. COPD a. predispose them to development of hypoxemia, sleep apnea, pulmonary hypertension, and ventilation disorders at even moderate altitudes. b. COPD is a risk factor for the development of AMS. c. Although oxygen saturation remains more than 90% in a healthy, awake individual until an altitude of 8000 feet, patients with COPD may desaturate below 90% at lower altitudes. High altitude increases hypoxic pulmonary vasoconstriction and may potentiate the development of cor pulmonale, which is known to adversely affect survival at sea level. Individuals with chronic COPD should be advised of the potential need for oxygen supplementation when traveling to moderate altitude, especially if they are already using oxygen at sea level or if dyspnea or fatigue becomes worse. Use of a pulse oximeter can guide the need for increased oxygen supplementation. 4. Asthma a. May actually do better at higher altitude - fewer allergens, pollutants b. Even those with exercise induced bronchospasm do not have worsening symptoms while exercising at 5000 feet. In addition, AMS incidence is not increased in asthmatics. People with asthma traveling to higher elevations should continue their usual medications and carry a rescue supply of bronchodilators and steroids. 5. Sickle cell disease a. At increased risk for pain and ischemic crises even at low altitude b. In patients with sickle cell disease, exposure to even low to moderate altitudes (4000 to 6500 feet) will provide additional hypoxia stress. Up to 20% of patients with hemoglobin sickle cell and sickle cell- thalassemia disease may experience a vaso-occlusive crisis, even under pressurized aircraft conditions. Oxygen is therefore advised for air travelers who have sickle cell disease. 6. Pregnancy a. an increased incidence of complications in maternal, fetal, and neonatal life. b. Lower birth weight, increased premature birth c. Increased risk of gestational hypertension, preeclampsia d. Travel above 13000 ft is not advised 7. Children a. Most healthy children do well, those at higher risk: i. < 6 weeks old, those with a history of pulmonary HTN, premature infants, children with Down syndrome, CHD, CF or neuromuscular problems b. Symptoms of AMS are difficult and non-specific in preverbal children c. Lower risk of HAPE < 2 yrs d. Symptoms and signs of most other high altitude illness approaches that in adults as the child gets older 8. Patients post radial keratotomy (shownotes) Patient with a history of radial keratotomy may experience hyperopic (farsighted) visual changes with ascent above 9000 feet. This results from corneal swelling from ambient hypoxia because the cornea is markedly sensitive to both systemic and ambient oxygen tension. In normal corneas, this swelling is uniform. After radial keratotomy, the swelling is exacerbated and inconsistent secondary to the pattern of the incisions. Photorefractive keratotomy and LASIK, which use laser techniques that do not produce incisions but instead shave the cornea and corneal stroma, respectively, do not result in similar problems.

Describe 5 options for prevention of altitude illness

1. Gradual / staged ascent that allows for acclimatization a. Limit sleeping height to less than 1000-1600 ft gain/day b. One extra night of acclimatization (at the same sleeping altitude) should be added for every 3000 to 5000 feet of altitude gain above 10,000 feet. 2. Excursions: "Climb high, sleep low approach" 3. Altitude pre-exposure regimens in hypoxic environments 4. Mild to moderate exercise at elevation 5. Hydration with balanced solutions (dilute, good urine output) 6. Pharmacologic prevention a. Acetazolamide for acclimatization in high risk groups b. Ibuprofen for headache prevention c. Dexamethasone for anti-nausea and mood enhancement 7. Oxygen Interestingly, you can also affect the relative altitude effects by watching the seasonal variations in barometric pressure. In the winter, barometric pressures tend to be lower making "relative altitudes" physiologically higher. Local weather can also significantly affect the barometric pressure. A low-pressure front can reduce the barometric pressure 12 to 40 mm Hg and so increase the "relative altitude" by 500 to 2500 feet. At extreme elevations this can be physiologically relevant.

List 5 DDx's for Osborne J-Waves

1. Local cardiac ischemia 2. Sepsis 3. CNS lesions 4. Hypercalcemia 5. Hypothermia

What is keraunoparalysis?

A finding in lightning strike victims: Charcot, in 1889, described the phenomenon termed keraunoparalysis, with the victim of a strike awakening to find himself or herself on the ground, unable to move the limbs. This flaccid paralysis, which is usually accompanied by marked vasomotor changes that result in extremities that appear blue, mottled, and pulseless, may persist up to 24 hours. The lower extremities are more commonly involved, and the typical pattern is recovery over minutes to days

What is core-temperature afterdrop?

A diminishment in the patient's core body temperature after removing them from the cold. This is caused by cool blood flowing to the core from the colder extremities as a result of vasodilation in the re-warmed patient. This results in greater drops in core temperatures and causes wide fluctuations in MAP and sudden change in peripheral vascular resistance NOTE: Avoid this by warming the core of a hypothermic patient first

List 5 potential pathologies associated with pulmonary barotrauma

A diver who takes a full breath at 33 fsw (feet below sea water), will have TWICE the volume of air in their lungs at the surface (according to Boyle's law). - A change in depth of as little as 3-4 ft can force air bubbles across the alveolar-capillary membrane - This volume expansion either causes the alveoli to rupture (pneumo) or the gas goes across the alveolar-capillary membrane to cause an air gas embolism *****the greatest risk for pulmonary barotrauma occurs in less than 10 feet of water***** Clinical entities to watch for: 1. Pneumothorax 2. Pneumomediastinum 3. Subcutaneous Emphysema 4. Alveolar hemorrhage Pearls: - Asthmatics: two fold higher risk due to: i. Narrow airways, bronchospasm and mucous plugs, reduction of breathing capacity with depth (100 fsw - 50% reduction), breathing of cool air can cause bronchospasm, compressed air can be contaminated with pollen. ii. Asthmatics have rules/regulations about safety to dive as a result - Risk factors: rapid ascent, panic, buoyancy regulation problems, insufficient air for the dive

What complications of asthma are associated with diving?

A patient with active asthma should also be strongly discouraged from diving even if they have normal pulmonary function tests, because individual lung units may have prolonged time constants, exhibit gas trapping, and rupture upon ascent. However, asthmatics at their baseline status with normal pulmonary function and easily controlled airway reactivity during exercise are probably at only a minimally increased risk of diving-related barotrauma. - Uptodate

Describe the management of DCS. What other diving disorders require recompression therapy?

ALL types of decompression illnesses require recompression - with the goal of washing out nitrogen, compressing air bubbles, deliver oxygen to ischemic tissue. Recompression therapy is the only definitive treatment for DCS and AGE, even if spontaneously resolved. Recompression therapy may still be helpful, even if delayed up to 10-14 days. Indications (See Box 143-2) - DCS I, II, - AGE - Delays of 5 hrs, lead to 50% mortality with AGE. - Contaminated air (causing CO poisoning. Problem with packaging the gas, rebreathers with soda lime dust contamination, etc. a. How would you manage a patient requiring recompression in the pre-hospital and ED environment (pre-HBO treatment)? While resuscitating in the ER: - Put the patient on 100% O2 - Follow the ACLS guidelines - Take a history and physical! Look for DCS Type I or II! - Resuscitate with IV fluids, to goal u/o of 0.5 ml/kg/hr. Don't over resuscitate people with pulmonary edema or AGE!! - Treat and avoid hypothermia - Positioning the patient in the left lateral decubitus (Durant's maneuver) and mild Trendelenburg (bed angled downward toward head) position in an effort to restore forward blood flow by placing the right ventricular outflow tract inferior to the right ventricular cavity, permitting air to migrate superiorly to a non- obstructing position White and black pearls.... - No benefits with ASA, steroids - Consider lidocaine IV for anyone with AGE and serious neurosymptoms - for neuroprotection - Cardiac arrhythmias may not respond to defibrillation until the patient is recompressed - Benzos for seizures - Fill the foley and the ET balloon with saline (not air)

How do carbonic anhydrase inhibitors prevent and treat acute mountain sickness?

Acetazolamide has myriad beneficial effects. By acting as a carbonic anhydrase inhibitor, it: - enhances renal bicarbonate diuresis and so improves renal correction of the ventilation-related respiratory alkalosis encouraging increased ventilation and arterial oxygenation. - It decreases nocturnal period breathing and so improves sleep. - It acts as a diuretic and so attenuates fluid retention common in patients with AMS. [although dehydration is risk factor for AMS] - It lowers CSF volume and pressure, which may play an additional role in its therapeutic effect. In addition, it has positive effects beyond its role as a carbonic anhydrase inhibitor, with beneficial chemoreceptor effects on ventilatory drive, alterations of cerebral blood flow, relaxation of smooth muscles, and upregulation of fluid resorption in the lungs. Adverse reactions: - Paresthesias - Polyuria - Nausea - Diarrhea - Drowsiness - Tinnitus - Myopia - Change of taste in foods - Sulfa medication cross reactivity

Describe 6 techniques for active rewarming

Active External Rewarming - Methods conduct heat with the skin - Options include: i. Plumbed garments: Warm fluids are pumped through malleable tubes that are in direct contact with the patient's skin - Hot water bottles - Heating pads - Forced-air warming systems (e.g., "bear hugger"). Air is pumped through a plastic blanket, transferring warmth directly to the patient - Radiant sources of heat Active Core Rewarming - As the name suggests, these methods of rewarming directly warm core body structures (e.g., airways, etc...) - Includes: i. Airway rewarming: Use of heated humidified air to warm the core and improve pulmonary functioning ii. Peritoneal dialysis iii. Heated irrigation (e.g., thoracic lavage). Warmed fluids are pumped into each hemithorax via thoracostomy tubes iv. Endovascular Rewarming: Use femoral vein closed circuit system to comatose or cardiac arrest patients v. Diathermy: Conversion of energy waves into heat. Use ultrasonic or microwave waves to deliver heat to the patient vi. Extracorporeal Rewarming (e.g., dialysis). Directly warming the blood by removing and heating it, then returning it to the patient. NOTE: In general, the active INVASIVE core rewarming methods are reserved for cases where there is a patient with moderate to severe hypothermia who fails to respond to traditional means, is requiring CPR and isn't easily transferred to an ECMO / CPB centre. NOTE: Very rapid rates of rewarming do not necessarily improve survival. Complications of rapid rewarming include DIC, pulmonary edema, hemolysis, and acute tubular necrosis. Extracorporeal circulation can provide cardiovascular support in perfusing but hemodynamically unstable patients. Extracorporeal rewarming should be considered in hypothermic cardiac arrest patients if there are no contraindications to CPR

What are the causes of hypoxemia?

Acute hypoxemia: - Low inspired Fi02 - Inadequate tidal volume (hypoventilation) - Diffusion limitation - V/q Mismatch - Right to left shunt Chronic mild hypoxemia causes a chronic mild acid load on the respiratory, renal, and blood buffer systems; acute illness such as a respiratory infection can rapidly cause a decompensation in this fragile balance, resulting in a worsening acidosis.

Describe common early symptoms of acute radiation syndrome (ARS). What clinical manifestations are predictive of severity of exposure and subsequent outcomes?

Acute radiation syndrome (ARS) occurs after a patient is exposed to whole body radiation. ARS can result from external or internal exposure to radiation and varies in nature and severity by dose, dose rate, dose distribution, and individual susceptibility. In general, cells that are undifferentiated, divide quickly, and have high metabolic activity are most radiosensitive. Examples of these types of cells include bone marrow stem cells, lymphocytes, spermatogonia, intestinal crypt cells, and epidermal basal cells. In general, the severity of ARS depends on the amount of radiation exposure (measured in grays) There are three phases to ARS: prodromal, latent, and manifest illness. 1. Prodromal - Nonspecific: anorexia, nausea, vomiting, and fatigue. - This phase is useful to help predict the severity of the radiation injury. - The presence, onset, and frequency of nausea and vomiting, although nonspecific, can serve as a prognostic factor. Early onset of nausea and vomiting, the persistence of it, and the presence of diarrhea indicates a severe radiation injury 2. Latent - initial symptom resolution - This step is skipped with people receiving a lethal dose of radiation 3. Manifest illness phase Hematopoietic sub-syndrome - the first sub-syndrome seen as the hematopoietic system is the most radiosensitive. - This sub-syndrome can appear at doses greater than 1 Gy and typically results in bone marrow suppression. At doses less than 1 Gy (100 rem), most cells survive but may be susceptible to radiation induced cancer. - Lymphocyte depletion is the first cell line to decrease and with high doses of radiation the drop will occur sooner and with greater severity. GI sub-syndrome - begins to occur at doses nearing 6 Gy about 1 week after exposure. Patients will display nausea, vomiting, gastrointestinal bleeding, malabsorption, and massive fluid loses potentially leading to hypovolemia and cardiovascular collapse. - These symptoms are due to death of the intestinal epithelial precursor cells and resultant denuding of the intestinal epithelial surface. Thrombocytopenia and immunosuppression from the accompanying hematopoietic sub-syndrome predisposes patients to infection and bleeding. - Neurovascular sub-syndrome - High doses of radiation, usually lethal - irritability, altered mental status, seizures, prostration, ataxia, and hypotension. Coma and death usually occur within a few hours. Because of the high dose of radiation needed to produce these findings, patients often die before progressing to the latent phase. The effects of radiation can be deterministic or stochastic. Deterministic effects are those in which the severity of injury is a function of dose (eg, bone marrow suppression). Stochastic or probabilistic effects are those in which the probability of an effect, rather than its severity, is a function of dose. An example of a stochastic effect is the development of radiation-induced cancer

What is the pathophysiology of AMS, HACE and HAPE?

All comes down to a combination of: inadequate adaptation both due to environmental pressures and personal genetic limitations. The big end-organs that fail - are the brain and the lungs. Hypobaric hypoxia's effects on central nervous system homeostasis give rise to AMS and HACE. AMS is the common, benign form that unheeded, can develop into rare, but potentially lethal HACE. AMS can develop within 4 to 8 hours of acute exposure to hypobaric hypoxia. HACE and HAPE typically occur 2 to 4 days after exposure to high altitude. It's the hypobaric hypoxia that triggers a complex pathophysiologic response...follow in Fig 136.2 Let's talk through the pathophysiology (truncated) for these: AMS → HACE - Exist on a spectrum - The definitive etiology of the cerebral forms of altitude illness remains unclear. Evidence suggests that clinical manifestations of AMS and HACE result from the combined effects of altered cerebral hemodynamics and inflammatory mediators that damage the blood brain barrier - Hypoxia: cerebral vasodilation occurs with increased arterial blood velocity and volume. - Hypocapnia: cerebral vasoconstriction - Impaired cerebral autoregulation - Systemic hypertension (from strenuous exercise) - Vasogenic edema from multiple mediators and damage to the blood brain barrier - Failure of CSF buffering HAPE: - HAPE results from overly exuberant increases in pulmonary arterial pressures (hypoxia-induced acute pulmonary hypertension) that lead to stress failures of the delicate pulmonary capillary beds → progressing to alveolar and interstitial edema. - It's this hypoxic pulmonary vasoconstriction response (HPVR) that leads to blood "backup" in the lungs. - There is a wide variation in both individual HPVR due to epigenetic reasons as well as UNEVEN vasoconstriction leading to patchy pulmonary edema. - Proposed reasons? (shownotes) i. decreased nitric oxide bioavailability at the pulmonary tissue level. ii. Acute inflammatory mediators post lung injury iii. Sodium channel-mediated alveolar fluid clearance changes (inhaled beta-adrenergic agonists, which have been proven to decrease risk of HAPE.) iv. Sensitized lung due to ongoing pulmonary infection See Figure 136.2 in Rosens 9th Edition, Chapter on High Altitude Medicine

List 4 types of radiation with potential for contamination

Anything that is ingested: - Alpha particles - Beta particles - Gamma particles - Radioisotopes that biodisposition into certain organs (e.g. radioiodine concentrating in the thyroid and resulting in thyroid cancer) The principles of radiation protection include time (reducing the time that you are exposed to radiation), distance (the intensity of radiation dose decreases inversely with the square of the distance), shielding (e.g. lead), and quantity (amount of a radioisotope in an area). The effectiveness of shielding varies with the type of the radiation. For example, alpha particles can be stopped by a thin piece of paper or even the dead cells in the outer layer of the skin, whereas thick, dense shielding is necessary to protect against gamma rays.

How do you calculate the alveolar partial pressure of oxygen?

As described by the alveolar gas equation, for any given inspired oxygen tension, the level of ventilation determines alveolar oxygen: as the Paco2 decreases, Pao2 correspondingly increases. The factors that determine the values for alveolar pO2 and pCO2 are: - The pressure of outside air - The partial pressures of inspired oxygen and carbon dioxide - The rates of total body oxygen consumption and carbon dioxide production - The rates of alveolar ventilation and perfusion -- From Wikipedia See Box 136.1 in Rosens Chapter 136 9th Edition Respiratory quotient = the ratio between the amount of CO2 produced in metabolism and the oxygen used (usually ranges from 0.8 - 1)

List 5 laboratory abnormalities expected in hypothermia

Blood Gases i. Increased PaO2 ii. Increased PaCO2 iii. Decreased pH Basic Hematologic Evaluation iv. High hematocrit v. Leukopenia vi. Thrombocytopenia vii. Increased Creatinine/BUN viii. Hyper/hypoglycemia ix. Elevated serum lipase

What are the indications for CPR, defibrillation, and antidysrhythmics in the hypothermic patient?

CPR indications: - Asystole - If no ECG available - No palpable central pulse after a ONE minute pulse check (note this is different than the standard BLS ten second pulse check) NOTE: Some experts recommended against providing CPR to patients with PEA arguing that the patients may have a small amount of cardiac output (without a palpable pulse) that would be disrupted with CPR. The goal in these cases is to WARM the patient up and thereby direct all human efforts to correcting the main underlying problem (i.e. not the pulselessness, but the hypothermia!). Defibrillation: - Not a lot of evidence to guide our management on this, so consensus seems to support: i. Trial of epinephrine 1 mg if in arrest (knowing that below 30 degrees it will not be very effective and may be prodysrhythmic) ii. Trial single defibrillation attempt at 360 J iii. If unsuccessful don't re-attempt until the patient has been warmed 1-2 degrees Celsius; then attempts may resume again. - Key point: don't make the patient with bradycardia go into VF! i. Avoid disturbing the moderate/severely hypothermic patient! ii. Avoid multiple attempts at defibrillation with 360 J for patients with temperatures < 28 degrees Celsuis and a wide complex tachyarrhythmia - there is no clear consensus on how many or how few shocks to give Antidysrhythmics: - Key goal is to rewarm the patient! - Most hypothermia-induced dysrhythmias convert spontaneously during rewarming. - Atrial dysrhythmias are common below 32°C (89.6°F), and are associated with a slow ventricular response. Atrial fibrillation is common but self-limited. It usually converts spontaneously during rewarming. Beta blockers and calcium channel blockers are contraindicated unless there is a rapid ventricular response. - No clear consensus on administering antiarrhythmics - probably safer not to. i. The ideal approach to ventricular dysrhythmias in the hypothermic patient has not been well studied. Lidocaine and propranolol have minimal hemodynamic effects during hypothermia. Their efficacy in the treatment of ventricular dysrhythmias appears limited. ii. The efficacy of amiodarone is not supported either iii. In hypothermia, at least one Group 1 antidysrhythmic agent, procainamide, increases the incidence of VF. Another drug in the same group, quinidine, can prevent VF during induced profound hypothermia and during cardiac manipulation at 25°C to 30°C (77°F-86°F). Bradycardia management: - Bradycardia is physiologic in hypothermia i. Don't need to start pacing unless the temperature is >32 degrees Celsius and the patient is hypotensive ii. Avoid transvenous approaches to unstable bradycardia - transcutaneous approaches are less irritating to the cold heart - Transvenous cardiac pacing is hazardous for bradydysrhythmias in hypothermia. i. External pacing may be worth trying in the rare setting of profoundly disproportionate bradycardia. Transcutaneous pacing has been used to facilitate continuous arteriovenous rewarming in perfusing patients by raising the systolic blood pressure above 60 mm Hg. Other active rewarming techniques do not require specific pressure gradients. Check out Dr. Doug Brown's video explaining a prehospital approach to accidental hypothermia: https://vimeo.com/57950513

Describe 3 CV manifestations of hypothermia

Cardiovascular responses to hypothermia are: - Tachycardia i. Occurs initially, followed by progressive bradycardia - Bradycardia i. The pulse diminishes with every drop in core body temperature ii. Occurs as the result of decreased spontaneous depolarization of the pacemaker cells in the heart ○ NOTE: This bradycardia will not respond to atropine - Osborn (J) Waves i. Look at the junction of the QRS complex and ST segment ii. Typically appears with a core body temperature below 32 degrees Celsius - Prolongation of PR, QRS, and QTc (think - everything gets cold and slows down) - Atrial Fibrillation i. Typically a rhythm that converts spontaneously after re-warming - Ventricular Fibrillation/Asystole i. Occur for a multitude of reasons in the hypothermic patient ii. Can occur spontaneously after the core body temperature falls below 25 degrees Celsius

List 8 sequelae of frostbite.

Categories = Neuropathic, MSK, Dermatologic, Miscellaneous Neuropathic 1. Pain i. Phantom pain ii. Complex regional pain syndrome iii. Chronic pain 2. Sensation i. Hypesthesia ii. Dysesthesia iii. Paresthesia iv. Anesthesia 3. Thermal sensitivity i. Heat ii. Cold 4. Autonomic dysfunction i. Hyperhidrosis ii. Raynaud's syndrome Musculoskeletal 1. Atrophy 2. Compartment syndrome 3. Rhabdomyolysis 4. Tenosynovitis 5. Stricture 6. Epiphyseal fusion 7. Osteoarthritis 8. Osteolytic lesions 9. Subchondral cysts 10. Necrosis 11. Amputation Dermatologic 1. Edema 2. Lymphedema 3. Chronic or recurrent ulcers 4. Epidermoid or squamous cell carcinoma 5. Hair or nail deformities Miscellaneous 1. Core temperature afterdrop 2. Acute tubular necrosis 3. Electrolyte fluxes 4. Psychological stress 5. Gangrene 6. Sepsis

List six mechanisms of lightning injury

Conduction injuries - Direct strike - Contact strike (hits something the person is holding on to or touching) - Side flash or splash injury - Streamer (hits the ground and then travels through the patient) i. May cause the "stride voltage injury" Flash / arc burns (current doesn't enter the body) Blast injury - Barotrauma - Shrapnel - Blunt trauma

List 8 DDx for hyperthermia/hyperthermia (↑temp and altered)

Differential diagnosis of heatstroke - CNS haemorrhage - Toxins, drugs - Seizures - Malignant hyperthermia - Neuroleptic malignant syndrome - Serotonin syndrome - Thyroid storm - High fever, sepsis - Encephalitis, meningitis Examples include - Cerebral malaria, typhoid fever, typhus - Graves' disease or thyroid storm Anticholinergic poisoning (note that they should have mydriasis*) - Overdose of sympathomimetics, stimulants (amphetamines, cocaine, PCP) - ASA or plavix overdose - Dietary supplement use in a hot environment - ephedrine or ergogenic aid creatine - NMS: muscle rigidity, severe dyskinesia or akinesia, hyperthermia, tachycardia, dyspnea, dysphagia, and urinary incontinence. - Serotonin syndrome: mental status changes, autonomic hyperactivity, and neuromuscular abnormalities secondary to increased CNS serotonergic activity (clonus). - Delirium tremens - Hypothalamic hemorrhage

What are the types of gas-mixture injuries?

Divers who dive to less than 130 feet usually just use compressed air; however when people go deeper they may use an artificial mixture of gas to extend the duration of their "bottom time" and avoid some of the disorders that happen at depth. Examples are nitrox, trimix, and heliox. The main challenge is either oxygen toxicity or hypoxemia due to the gas blend ratios and dive profile. In Nitrox the percentage of oxygen in the mix is increased, reducing the nitrogen fraction. This means that there is less nitrogen uptake at a given depth.]The critical caveat with nitrox is that its higher oxygen content means that a diver breathing nitrox is at risk of developing oxygen toxicity at a shallower depth than a diver breathing air. Therefore, these folks are at risk for oxygen toxicity. See: https://www.diversalertnetwork.org/medical/faq/Breathing-gas-mixture There can also be major risks when mixing the gases - explosions, fires, asphyxiation, toxic additive additions (CO poisoning), hydrocarbons and others.

List clinical findings (early and late) associated with electrical injuries.

Early: go head to toe CNS - Apnea, LOC, amnesia, peripheral nerve damage/paralysis, keraunoparalysis Cardiovascular - Asystole (DC or lightning) - Dysrhythmias (VF post AC current) - Third spacing (burns) - Thrombosis or vasospasm (MI is very rare) Resp - Respiratory muscle (and centre) paralysis! GI - Pancreatitis, solid organ injury, hepatitis, rhabdo MSK - Fracture, dislocation, rhabdomyolysis, compartment syndrome Skin - Burns - ***low external injuries DOES NOT mean low internal injuries*** - Deep tissue burns Late: go head to toe CNS: - Apnea, sequelae from direct cranial trauma, amnesia/LOC, seizures, cognitive dysfunction, anxiety, psychiatric effects - depression, sleep disturbances, nightmares, nocturnal enuresis, and separation anxiety. Hysterical blindness, deafness, and muteness have been observed. - hypoxic encephalopathy, intracerebral hemorrhage, cerebral infarction, and spinal fractures EENT - Cataracts CV - Thrombosis or vasospasm - Delayed arterial thrombosis as well as aneurysm formation and rupture have been reported following electrical injury and are due to medial coagulation and necrosis - Labial artery hemorrhage MSK - Compartment syndrome Skin Wound infection / scarring /contractures Psych - Anxiety - Panic disorders - PTSD - Depression

What are the types of ionizing radiation that cause injury?

Electromagnetic radiation: - Far ultraviolet (this is blocked by the ozone layer) - X-rays Particle radiation - Neutron radiation - Alpha particles - Beta particles - Gamma rays (photons) - that penetrate much deeper Red = generally dangerous with internal exposure Gamma and x-rays are high-energy photons that differ in their place of origin: gamma rays are emitted from the nucleus, whereas x-rays are produced as the result of changes in the positions of electrons orbiting the nucleus.

Describe the prehospital management of electrical injuries

Ensure scene safety **have the power company turn off the power to the line before rescuing anyone!** - Ensure PPE - Rapid triage for most critical victims (are there any other injured?) - ACLS care for anyone unresponsive - "Reverse triage" i. Priority are those people with no respiratory effort (respiratory muscle paralysis can last minutes) ii. PATIENTS WITHOUT SIGNS OF LIFE ARE TREATED FIRST - Cardiac monitoring and treatment of arrhythmias - Volume resuscitation - Rapid transport to the nearest hospital A COUPLE OTHER HOSPITAL MANAGEMENT PRINCIPLES: - Serum CK-MB measurements and ECG changes are poor measures of myocardial injury following electrical trauma; cardiac telemetry will more accurately assess for arrhythmias and autonomic dysfunction - Parkland and similar formulas used for fluid resuscitation following thermal burns should not be used in victims of electrical injuries, since surface burns may grossly underestimate the extent of injury. - Follow-up with ophthalmology is prudent to watch to delayed cataract formation

List 5 potential injuries in scuba diving other than Dysbarism

Environmental exposures - Hypothermia - Sunburn - Trauma (harpoon injury) Aquatic exposures: - Submersion accidents (drowning) - Motion sickness - Marine envenomations (see Ch. 61 - box jellyfish, blue octopus, rockfish) All the other dysbarisms and barotraumas are due to rapid pressure-volume changes in airfilled cavities or nitrogen dissolved into body tissues coming out...

What doses are generally fatal, potentially fatal (and produce significant symptoms) and are generally safe (minimal symptoms if any)?

Fatal: > 10 Gy Possibly fatal: > 5 Gy Generally safe: < 2 Gy See Table 138.2 (Question 6)

List aetiologies of hyperthermia

Fever = your body raises the set-point "the thermostat increases" - Elevated levels of prostaglandin E2 (PGE2) in the hypothalamus appear to be the trigger for raising the set-point. Once the hypothalamic set-point is raised, this activates neurons in the vasomotor center to commence vasoconstriction and warm-sensing neurons to slow their firing rate and increase heat production in the periphery. - Hyperthermia = your thermostat is set at 36 degrees, but your house is on fire - Despite your body trying to lose as much heat as possible, it is unable to dissipate the heat or some external factor is overwhelming the system. Aetiologies - Environmental • Heat stroke - Exertional - Classic • Excited delirium - Endocrine: • Hyperthyroidism - Drugs** • Ecstasy • Serotonin syndrome • Malignant hyperthermia • ASA overdose • Iron overdose • DNP Although the vast majority of patients with elevated body temperature have fever, there are a few instances in which an elevated temperature represents hyperthermia. These include heat stroke syndromes, certain metabolic diseases, and the effects of pharmacologic agents that interfere with thermoregulation. In contradistinction to fever, the setting of the thermoregulatory center during hyperthermia remains unchanged at normothermic levels, while body temperature increases in an uncontrolled fashion and overrides the ability to lose heat. Exogenous heat exposure and endogenous heat production are two mechanisms by which hyperthermia can result in dangerously high internal temperatures. - Rosen's 9th Edition, Chapter 119

Describe the management of a pregnant patient (1st trimester and 2nd /3rd trimester) in the setting of electrical injury.

For obstetric patients, the overall risk to the fetus is small, but a spontaneous abortion can occur. Secondary trauma may lead to placental abruption. Obstetric consultation and fetal monitoring are essential.

What are the major types of cold injuries?

Freezing and NON-freezing! These injuries can occur together, especially in climates that hover around 0 degrees.

What are the causes of tissue hypoxia?

From UpToDate Cellular hypoxia is a state in which there is insufficient oxygen to meet the metabolic demands of a given tissue. It can result from impaired perfusion (ischemia) and/or diminished arterial oxygen content (due to anemia or hypoxemia). Cellular tolerance of hypoxia is variable. As examples, skeletal muscle cells can recover fully after 30 minutes of hypoxia, but irreversible damage occurs in brain cells after only four to six minutes of similar hypoxic stress. Therefore, life-threatening hypoxemia needs to be treated with the administration of oxygen (and sometimes with red cell transfusion) while measures are being initiated to treat the primary cardiopulmonary insult. Cellular mechanisms that contribute to hypoxic cell injury include depletion of ATP, development of intracellular acidosis, increased concentrations of metabolic by-products, generation of oxygen free radicals, and destruction of membrane phospholipids. There is also a dramatic increase in intracellular calcium concentration, contributing to cellular injury via a variety of mechanisms, including direct damage to the cytoskeleton and induction of genes that contribute to apoptosis. Hypoxia also induces an inflammatory reaction characterized by neutrophilic infiltration, thus augmenting cellular damage via release of cytokine mediators, oxygen free radicals, and by intensifying ischemia due to disruption of the microcirculation.

If there are limited resources in mass casualty, how can triage of radiation victims be divided?

Given the limited resources casualties can be divided by: - Time to nausea, vomiting, diarrhea - Those with near immediate gastrointestinal symptoms have likely suffered a fatal dose (e.g. if time to vomiting is < 1 hour) - Patients who experience vomiting within 2 hours of exposure will require hospitalization and careful medical observation, because they are likely to have sustained life-threatening doses of radiation. - Development of neurovascular sub-sydrome within the first 24 hours should be provided comfort care measures because they likely were exposed to a lethal amount of radiation

How do you treat HACE?

HACE is the least common but most severe form of high-altitude illness. Death from HACE at as low as 2500m is reported, although most cases occur above 3000m. Mild AMS can progress to severe HACE with coma in as few as 12 hours. Although severe symptoms usually develop within 1 to 3 days, they may not occur until 5 to 9 days. Treatment: - DESCENT - High fio2 oxygen - Dexamethasone 8 mg IM or PO, then 4 mg q6 hrs. - ABCD's i. Some may require intubation ii. Management of increased ICP in extreme cases Early treatment of HACE generally results in good outcomes, but after coma is present, the mortality rate exceeds 60%. See HACE DDx in Box 136.7 Rosens 9th Edition

Compare exertional and classic heatstroke

Have different presentations and manifestations. - CHS occurs during periods of sustained high ambient temperatures and humidity, such as during summer heat waves. Victims are often older adults and poor and live in underventilated dwellings without air conditioning. Debilitated patients who have limited access to oral fluids may develop water depletion heat exhaustion, which progresses to heatstroke if untreated. Victims of CHS commonly suffer from chronic diseases, alcoholism, or schizophrenia, which predisposes to heat illness. Such patients are often prescribed medications (eg, diuretics, antihypertensives, neuroleptics, anticholinergics) that impair the ability to tolerate heat stress. Sweating ceases in most CHS patients. Factors such as advanced age, hypotension, altered coagulation status, and the necessity for endotracheal intubation on arrival at the ED predict a poor outcome, despite successful cooling measures. In contrast, patients with EHS are usually young and healthy individuals whose heat-dispelling mechanisms are overwhelmed by endogenous heat production. Athletes and military recruits are typical victims. Rhabdomyolysis and acute renal failure, rarely seen in patients with CHS, are common in patients with EHS. Sweating is present in 50% of cases of EHS. Hypoglycemia may occur as the result of increased glucose metabolism and hepatic damage, resulting in impaired gluconeogenesis. Coagulopathy is common; - Rosen's 9th Edition, Chapter 133

Describe the diagnostic features of heatstroke

Heatstroke: diagnosis (Box 133.5) i. Exposure to heat stress, endogenous or exogenous ii. Signs of severe CNS dysfunction (coma, seizures, delirium) iii. Core temperature usually >40.5C, but may be lower iv. Hot skin common, and sweating may persist - Marked elevation of hepatic transaminase levels - *Any type of CNS dysfunction can herald heat stroke: bizarre behaviour, opisthotonos, hallucinations, rigidity, cerebellar dysfunction - *Hepatic damage is consistently featured in heatstroke. - Look for elevated AST and ALT in the patient with altered mental status

List 5 expected injury patterns for high-voltage and lightning injuries

High voltage: Direct or arc injury patterns (burns / blunt trauma / cardio-respiratory arrest) Lightning: Injury occurs from the force of a strike, blunt trauma effects when the victim is thrown, the superheating of metallic objects in contact with the patient, blast-type effects and barotrauma, or shrapnel. High voltage: - Skin findings / burns (see Question 4) - Cardiac i. Arrest - Due to induced VF or asystole ii. Arrhythmias - Bradycardia, tachycardia. A fib, ectopy - Respiratory arrest i. Tetanic paralysis of thoracic respiratory muscles - Causing apnea - CNS: i. Direct injury to brainstem respiratory centres ii. Seizure disorder iii. Secondary vascular injury (stroke/CVT) from blood vessel injury iv. Vertigo v. Delayed and chronic manifestations include ascending paralysis, transverse myelitis, and amyotrophic lateral sclerosis. Peripheral neuropathies are a common result, most often involving the median and ulnar nerves. - Neuropsychiatric sequelae include anxiety, depression, mood lability, difficulty concentrating, and insomnia. These may become a persistent source of disability. - HEENT: i. Early cataract formation ii. vitreous and anterior chamber hemorrhages, retinal detachment, macular lacerations, and corneal or conjunctival burns. - Extremities i. Vascular injury - thrombosis or aneurysm formation leading to tissue ischemia /necrosis ii. Muscle necrosis iii. Vascular / smooth muscle paralysis and DVT formation iv. Tissue edema v. Compartment syndrome vi. Periosteal burns / necrosis vii. Fracture or dislocations from tetanic contractions or sudden myoclonic jerk at time of direct contact - Viscera: i. muscle damage may result in significant myoglobinuria, subsequent renal failure, and life-threatening hyperkalemia. These complications are more likely in patients who are hypotensive or volume-depleted. ii. Stress ulcers are a common gastrointestinal complication. iii. Uncommon but severe intra-abdominal injuries include a ruptured hollow viscus and necrosis of the pancreas or gallbladder. Pulmonary edema is rare. Although early deaths are due to respiratory and cardiac arrest, late deaths occur from sepsis, pneumonia, and renal failure. Lightning: - Skin findings / burns / fern-like (Lichtenberg) figures (see question 8 below) - CV system i. Asystole (progressing to ischemic cardiac and cerebral injury if the respiratory centre has been reactivated) - VF ii. Myocardial contusion iii. ECG changes (see wisecracks) - Resp system i. Respiratory muscle paralysis - CNS i. Apnea, due to effects on the medullary respiratory center, may persist for several hours. ii. Direct trauma may result in skull fractures, intracerebral and extracerebral hematomas and hemorrhages, cerebral edema, and elevated intracranial pressure. Leading to herniation and death. Cerebellar ataxia, peripheral nerve damage iii. transient loss of consciousness, amnesia for the event, and transient paresthesias and paralysis of the extremities. - Ears i. Rupture of TM due to shock wave / blast effect (expansion effect from air) ii. Hearing loss, tinnitus, vertigo, - Eyes i. Immediate or delayed onset of cataracts ii. Paralysis of ciliary muscle ● Other injuries - Due to blunt trauma or blast injury ....we'll re-summarize this in the next question!

Describe the physiologic process of acclimatization to high altitude. List a least three responses.

Hypobaric hypoxia stimulus leads to your body working hard to improved oxygenation. Acclimatization is both immediate (within minutes the carotid bodies sense hypoxemia) and continuous over months (hemoglobin increases may continue over more than 6 weeks). It involves multiple systems from protein synthesis to respiratory, cardiovascular, renal, and hematologic responses. Acclimatization begins as the oxygen saturation of arterial blood falls below sea-level values. The altitude at which this occurs depends on the rate of ascent, the duration of exposure, and the individuals physiology e.g. preexisting disease that limits cellular oxygen delivery (such as cardiomyopathy) or decreased pulmonary reserves. Here's the list: Acute (minutes to hours): 1.1 Increase in minute ventilation = hypoxic ventilatory response (HVR) - Within minutes of exposure to high altitude, the peripheral chemoreceptors in the carotid bodies sense the decrease in the partial pressure of oxygen in alveolus (Pao2) and signal the respiratory control center in the medulla to increase ventilation. - The magnitude of the HVR varies among individuals and may be genetically predetermined. HVR may also be inhibited or stimulated by numerous factors, including ethanol, sleep medications, caffeine, coca, prochlorperazine, and progesterone. 1.2 Catecholamine response to acute hypoxia: - increased cardiac output and elevations in heart rate, stroke volume, blood pressure, and venous tone. Except at extreme altitudes, acclimatization over weeks results in the gradual return of the resting heart rate to near sea-level values. Continued resting tachycardia is evidence of poor acclimatization. 1.3 Release of erythropoietin (in hours: but the response is delayed) - Leading to new circulatory red blood cells in 4 or 5 days. During the next 2 months, red blood cell mass increases in proportion to the degree of hypoxemia Delayed (days to weeks) 1. Renal excretion of bicarbonate to adapt to the respiratory alkalotic state induced by the HVR, Maximum rate/amount by 6-8 days 2. Hematopoietic responses - Increase in hemoglobin - Increase in number of red blood cells - Increase in MCV - Increased plasma volume - Increase total blood volume Other responses (shownotes) Hypoxemia also results in an increase in 2,3-diphosphoglycerate, causing a rightward shift of the oxyhemoglobin dissociation curve, which favors a release of oxygen from the blood to the tissues. This is counteracted by the leftward shift of the oxyhemoglobin dissociation curve caused by the respiratory alkalosis from hyperventilation. The result is a net null change in the oxyhemoglobin curve and an increase in oxygen-hemoglobin binding in the lung, which raises Sao2. Some individuals with mutant hemoglobin and high oxygenhemoglobin affinity are found to acclimatize more efficiently than their normal counterparts at moderate altitudes.

List 5 cooling measures for heatstroke + 3 adjuncts to therapy

Immediate cooling is the cornerstone of treatment. Patients who present to the hospital with heatstroke have high mortality rates ranging from 21% to 63%, and mortality increases significantly when cooling is delayed. -Rosen's Chapter 133 Key management: (one of two options!) 1. Evaporative cooling a. Strip all clothing, spray tepid water on patient, have fans blow air continuously over the patient 2. Immersion ice water cooling = fastest way of dropping a person's body temperature (10-40 mins) a. Adds challenges to resuscitation! Stop cooling once the temperature reaches 39 deg. There are some other "adjuncts" to the cooling process, but aren't as effective at dropping the body temperature rapidly - which is why they are call adjunctive! - Application of ice packs to high heat transfer areas (eg, neck, groin, axillae) is commonly used. - Cooling blankets may be a useful adjunct but will not produce rapid cooling if used exclusively. - Cold irrigant gastric or rectal lavage will not provide significant heat exchange if used as the primary cooling modality. In addition to cooling, there are other adjunctive therapies that need to be performed for these very sick patients with heat stroke: - Resuscitation of the ABC's i. Securing the airway (aspiration risk, control oxygenation and ventilation) ii. Targeted volume resuscitation. Some patients are profoundly hypotensive and volume deplete, while others can have right sided heart failure or pulmonary edema - so reassess and resuscitate prn iii. Hemodynamic resuscitation: focused on cooling. - A variety of tachyarrhythmias commonly occur during heatstroke. These usually resolve with cooling, and electrical cardioversion should be avoided until the myocardium is adequately cooled. The use of α-adrenergic agents such as norepinephrine is not recommended because they promote vasoconstriction without improving cardiac output or perfusion, decrease cutaneous heat exchange, and may exacerbate ischemic renal and hepatic damage. Atropine and other anticholinergic drugs that inhibit sweating should be avoided. - Treat metabolic complications: i. NO acetaminophen or ASA ii. Maintain urine output 2 ml/kg/hr, especially with rhabdomyolysis iii. IV benzodiazepines for excessive shivering or seizures Antipyretics are NOT on this list - they only work with febrile illnesses and NOT environmental illness Cooling modalities to lower body temperature in heatstroke - Preferred i. Evaporative cooling with large circulating fans and skin wetting ice waterimmersion - Adjuncts i. Ice packs to axillae and groin ii. Cooling blanket iii. Peritoneal lavage (unproven efficacy in humans) iv. Rectal lavage v. Gastric lavage vi. Cardiopulmonary bypass

What is immersion syndrome?

Immersion syndrome refers specifically to syncope resulting from cardiac dysrhythmias on sudden contact with water that is at least 5°C lower than body temperature. The risk is proportional to the difference between body temperature and water temperature. Wetting of the face and head before entrance into the water may prevent the inciting sequence of events. Mechanisms for the syndrome are vagal stimulation leading to asystole and ventricular fibrillation secondary to QT prolongation after a massive release of catecholamines on contact with cold water. The resultant loss of consciousness leads to secondary drowning. Undetected primary cardiac arrhythmia (may be a more common cause of drowning than generally appreciated). As an example, cold water immersion and exercise can cause fatal arrhythmias in patients with the congenital long QT syndrome type 1. Similarly, mutations in the cardiac ryanodine receptor (RyR)-2 gene, which is associated with familial polymorphic VT in the absence of structural heart disease or QT prolongation, have been identified in some individuals with unexplained drowning. - Uptodate

What are the targets of cooling?

In heat stroke the goal is to get to 39 degrees C. Once you reach that target - we stop cooling so we don't "overshoot" and make the patient hypothermic.

How can radiation injury occur?

Like other forms of energy injury, radiation exposure can be external (eg, exposure to x-rays or a radiation burn to the hand) or internal, resulting from the inhalation, ingestion, or injection of radioisotopes. The four different but interrelated units for measuring radiation (radioactivity, exposure, absorbed dose, and dose equivalent)

List 5 variables that portend poor outcome

List from Rosen's 1. Age - Very young LESS than 3 YEARS OLD (increased risk for hypothermia and more metabolic demands) - Very old (comorbidities) 2. Water temperature - Cold-water immersion speeds the development of exhaustion, altered consciousness, and cardiac dysrhythmia. 3. Duration and degree of hypothermia 4. Lack of diving reflex 5. Ineffective response to resuscitation efforts 6. Drug and alcohol use (cited in PMID 24607870 The list from Uptodate and Rosen's: - Duration of submersion >5 minutes (most critical factor) - Time to effective basic life support >10 minutes - Resuscitation duration >25 minutes - Age >14 years - Glasgow coma scale <5 (ie, comatose) - Persistent apnea and requirement of cardiopulmonary resuscitation in the emergency department - Arterial blood pH <7.1 upon presentation Break it down into: - Event factors (duration, time to BLS) - Patient factors (age (older), drug and etoh use, comorbidities) - Clinical factors (hypothermia, long resuscitation duration, low GCS, ongoing cardio/pulmonary arrest (especially asystole), low pH, unreactive pupils) Neurologically intact survival is reported for individual patients even with several of these factors present; none of several proposed scoring systems using combinations of these variables has 100% predictive power. Children who present with an abnormal head computed tomography (CT) scan (eg, intracranial bleed, cerebral edema) within the first 24 hours have a nearly 100% mortality rate. Furthermore, an abnormal head CT scan at any time is associated with poor outcome (death or persistent vegetative state). Adverse neurologic findings on initial presentation do not preclude full neurologic recovery, although in general, patients whose duration of submersion or resuscitation exceeds 10 minutes have an unfavorable outcome. Unfortunately, in reality we never really know these details accurately when that resuscitation rolls through the doors...

How do you treat acute mountain sickness?

Management of AMS must adhere to the axiom, "After the symptoms of altitude illness occur, further ascent to a higher sleeping altitude is contraindicated." SEE ROSENS BOX 136.2 9th Edition 1. Mild a. Do NOT ascend to higher sleeping altitude b. Wait for symptom resolution and acclimatization (usually 3-4 days) c. Consider pharmacology 2. Moderate a. Rest b. Descent (even 150m makes a difference!) c. Pharmacology i. Aspirin, ibuprofen, and acetaminophen are useful for the treatment of high-altitude headache. 1. Narcotic analgesics should be avoided because of depression of the hypoventilation response (HVR) and respiratory drive during sleep. ii. Antiemetics 1. for nausea and vomiting, prochlorperazine unlike other antiemetics, stimulates the HVR. iii. Acetazolamide = respiratory stimulant (prevents periodic breathing which worsens insomnia) 1. Also enhances renal bicarbonate diuresis, improves 2. 62.5 mg - 125 mg BID a. Avoid Benzo's or other respiratory depressants especially alcohol iv. Consider dexamethasone 3. Severe a. Rest b. Descent c. Pharmacology i. As above ii. Consider adding: 1. Dexamethasone 8 mg po; then 4 mg po q6hrs for 3 days a. Euphoric effects b. anti-inflammatory properties, possibly to reduce cerebral blood flow, and to block the action of vascular endothelial growth factor. d. Oxygen / hyperbaric oxygen therapy

List 5 mechanisms of heat loss and 5 physiological responses to cold

Mechanisms by which heat is lost are: -Radiation i. Most profound mechanism of heat loss at rest (>50%) - Conduction i. At rest, not very much heat is lost ii. If wet, increases dramatically -Convection i. Works with conduction to increase heat loss by up to 25x in the wet patient - Respiration - Evaporation NOTE: Conduction, convection, and radiation are the biggest mechanisms by which heat is lost in the outdoors: it's key to develop behavioral defenses against these - Physiologic responses to cold are: i. Shivering ii. Increased pre-shivering muscle tone iii. Vasoconstriction iv. Non-shivering basal thermogenesis v. Endocrinologic thermogenesis

Define moderate altitude, high altitude and severe altitude

Mod - 1500 m - 2400 m (5k - 8k ft) - Rapid ascent to this altitude may result in mild, transient symptoms, but severe altitude illness is uncommon. High - 2400 m - 4200 m (8k - 14k ft) - High altitude illness is common with rapid ascent to this height, especially in anyone with pre-existing medical illness Severe - 4200 m - 5400m (14k - 18k ft) - High risk for altitude illness with rapid ascent - including the severe forms: HACE and HAPE Extreme altitude - 5400m (18k ft and up) -Although climbers using careful acclimatization schedules can transiently tolerate this height, complete acclimatization generally is not possible and long visits above this level result in progressive deterioration. Given limitations in physiologic reserves, climbers who become incapacitated at this elevation typically are dependent on others to survive

What are the changes are made to BLS and ACLS in the setting of hypothermia?

NOTE: Don't forget the C-A-B's. Know that patients who are cold will have bradycardia - so have multiple people feel for a central pulse for at least one minute or use bedside echocardiography. NOTE: If there are signs of life present (coordinated electrical activity on ECG; pulse palpable; cardiac contraction on echo) CPR should be withheld. Experts suggest that the changes to standard ACLS apply to patients < 30 degrees Celsius. Once the patient is re-warmed to > 30 degrees Celsius, normal protocols apply. According to Dr. Doug Brown's paper: The guidelines of the European Resuscitation Council recommend a modified approach to advanced life support, consisting of up to three defibrillations, with epinephrine withheld until the core temperature is higher than 30°C (86°F) and with the interval between doses doubled until the core temperature is higher than 35°C (95°F). These recommendations conflict with the American Heart Association guidelines, which state, "It may be reasonable to consider administration of a vasopressor during cardiac arrest according to the standard ALS [advanced life support] algorithm concurrently with rewarming strategies." Hence, the administration of up to three doses of medication and defibrillation is likely to be a reasonable approach, with further dosing guided by the clinical response.

In what situations would you not initiate resuscitation of a hypothermic patient?

NOTE: Remember you can be cold and dead....starting a resuscitation that is NOT medically indicated puts many people at risk (not to mention uses resources ineffectively!) Do not resuscitate (do CPR) the patient if: 1. DNR signed and present 2. Obvious lethal injuries are present 3. Major signs of blunt/penetrating trauma 4. Snow-packed airway 5. Chest wall compression is impossible 6. Abdomen is rigid (frozen) 7. No signs of life and asystole on the ECG (judgement call depending on the duration of burial (e.g. <35 min) and "down time" 8. Retrieval or resuscitation places the rescuers at too much risk 9. Too cold (? less than 9-15 degrees C) - controversial 10. Has signs of life (pulse, moving, respiratory effort)

Define mild, moderate and severe hypothermia

NOTE: Remember you need to use a CORE temperature probe - so go rectal or bladder or preferably esophageal. The stages of hypothermia are as follows: Mild Hypothermia - 35-32 degrees Celsius Moderate Hypothermia - 32-29 degrees Celsius Severe Hypothermia - 28-22 degrees Celsius Profound Hypothermia - 20-9 degrees Celsius - 9 degrees Celsius is defined as the lowest therapeutic hypothermia survival level Note: There is a five level "Swiss staging" system that is used in some places (also supported by the International Commission for Mountain Emergency Medicine). This system is used by the International Commission of Alpine Rescue (ICAR). However, it's not favored by all experts in the field of wilderness medicine because the clinical (especially the neurological) symptoms of hypothermia range widely from person to person. For example, a person could still be shivering and have a temperature below 32 deg Check out: http://www.alpine-rescue.org for more.

Describe the CNS, CVS, hematologic, and GU presentations associated with hypothermia for each stage (mild, moderate, severe)

NOTE: See table 132.1 in Rosen's 9th Edition for a complete table detailing the various manifestations of hypothermia Mild Hypothermia - 35-32 degrees Celsius - CNS i. Amnesia ii. Dysarthria iii. Ataxia iv. Apathy - CVS i. Normal blood pressure maintained - Heme i. No effects - GU i. Urine temperature drops Moderate Hypothermia - 32-29 degrees Celsius - CNS i. Stupor ii. Decreased LOC iii. Pupils dilated - CVS i. Atrial fibrillation ii. Pulse and cardiac output drop to 2/3 of normal - Heme i. Insulin ineffective Severe Hypothermia - 28-22 degrees Celsius - CNS i. Loss of reflexes and voluntary motion ii. No response to pain - CVS i. Ventricular fibrillation susceptibility ii. Cerebral blood flow 1/3 normal iii. Cardiac output 45% of normal iv. Pulmonary edema susceptibility v. Significant hypotension - Heme i. Major acid-base disturbances

Discuss the indications for rewarming the drowning patient. To what temperature do we warm to?

No consensus exists with regard to the appropriate length of resuscitative effort for hypothermic drowning victims in the ED. The safest parameter is to continue until the core temperature reaches at least 32°C to 35°C, because cerebral death cannot be diagnosed accurately in hypothermic patients with temperatures below this level. This parameter may not always be practical, however, because brain-dead patients are often poikilothermic.

List 10 predisposing factors for heat illness

Non-exertional heat illness increase the risk for classic heatstroke during periods of high heat and humidity: - advanced age, - psychiatric conditions, - chronic disease, - obesity, - medications - Pump or circulation problem: i. Beta blockers, CCBs - Dehydration i. Diuretics - Inhibition of sweating i. Anticholinergics, stimulant drugs Exertional heat illness: - Lack of acclimatization - Inadequate hydration (cool, flavored liquids) - goal body weight within 1% of previous day's weight - Weight class / category athletes - Wet bulb globe temperature > 25 Including discussion of heat illness pathophysiology: Car analogy. Coolant (blood) is circulated by a pump (heart) from the hot inner core to a radiator (skin surface cooled by the evaporation of sweat). Temperature is sensed by a thermostat (CNS), which alters coolant flow by a system of pipes, valves, and reservoirs (vasculature). Failure of any of these components can result in overheating. (See Rosen's 133.2 for a visual representation of this analogy)

Describe memory aids for the gas laws

Pascal's law - Tube of Toothpaste: A pressure applied to any part of a liquid is transmitted equally throughout. Boyle's law - Syringe aspirating fluid: at work you draw up local anesthetic or any drug by withdrawing a plunger on a syringe (decreasing the pressure inside the chamber and increasing the volume of the chamber) thereby creating an area of less negative pressure and more volume for the drug or liquid to enter! The incremental changes in pressure (and therefore volume) are greatest at the surface, so barotrauma most commonly occurs near the surface (other at the beginning or at the end of a dive). Long or deep dives are not required for barotrauma. Henry's law - Carbonated soft drink analogy: An everyday example is given by one's experience with carbonated soft drinks, which contain dissolved carbon dioxide. Before opening, the gas above the drink in its container is almost pure carbon dioxide, at a pressure higher than atmospheric pressure. After the bottle is opened, this gas escapes, moving the partial pressure of carbon dioxide above the liquid to be much lower, resulting in degassing as the dissolved carbon dioxide comes out of solution. - Wikipedia

Differentiate between active and passive rewarming. What are the two types of active rewarming?

Passive Rewarming - Non-invasive form of reheating; the patient should have the capacity to spontaneously rewarm themselves - Involves covering the patient with an insulator (blanket), and minimizing heat loss by evaporation and convection - Should have a warm room temperature, preferably >21 degrees Celsius - Remember, the primary means by which these individuals will reheat themselves is shivering; if they are able to do that (generally core temperature >32 degrees Celsius), passive rewarming is appropriate - Rates of passive rewarming are not set in stone, so look for a rise of 0.5-2.0 degrees Celsius per hour - You need to rewarm them fast enough to limit exposure to life-threatening arrhythmias - To recap, the key steps: i. Remove the patient from the cold ii. Remove any wet clothes and dry the patient off iii. Cover them up with clothes iv. Turn up the temperature in the room Active Rewarming - Direct transfer of heat to the patient by invasive/non-invasive means - Used primarily for moderate to severe hypothermia - Required for moderate to severe hypothermia where cardiovascular instability is present - Defibrillation is not as successful when the patient is hypothermic Two types of active rewarming are: - Active External Rewarming - Active Core Rewarming

Describe prehospital and emergency department management of the radiation victim.

Prehospital: - Activate / call your local disaster plan expert / administrator - Information should be gathered regarding the exposure event: the numbers and types of patients potentially affected, the radionuclide involved, the route of exposure, and the estimated dose of radiation. - Treat immediate life threats! - Unstable patients should be rapidly transported in lieu of decontamination measures. Radio contact with the receiving hospital should be provided to facilitate preparations. If the community disaster plan has a designated hospital for radiation-contaminated victims, patients should be transported directly to that facility, bypassing hospitals less equipped to care for these complicated patients. - Decontamination should be initiated at the scene. Patients with abnormal vital signs should have partial decontamination, such as clothing removal, at the scene before expeditious transportation to an ED. ER: 1. Preparation - Hopefully there has been some prehospital notification and activation of the disaster process i. Number of patients and types of injuries ii. Roles clearly assigned - "I FLOP approach" iii. Radiation officer to minimize spread of contamination iv. Media relations person - Decontamination area 2. External contamination management: - Radiation contamination is not an acute threat to the life of the patient or the provider, and its presence should not preclude institution of lifesaving measures. If standard precautions are taken, the risk to the health care providers is minimal. - E.g. of Alexander Litvinenko - 3 weeks of health care worker contact and no harms reported to the HCWs - Think of it like the person who just had a CT scan and was "irradiated" - 90% of the radiation from the patient can be removed by taking off their clothing and shoes and placing them in plastic bags - soap and water to wash down skin will help further. High pressure, repeat cleaning methods may be required - Universal precautions, including rubber gloves, shoe covers, and respirators if airborne contamination is suspected, are effective in protecting personnel and the work area from contamination. The only variation is to wear two sets of gloves and to change the outer pair when appropriate to avoid cross-contamination. - Assess for Local and Acute Radiation syndrome - Local radiation injury = treat like a burn and refer to a burn centre within 72 hrs: Due to the chronic vascular injury and the potential for even minor trauma to the area to recapitulate the injury, the following are important in the treatment of LRI: topical corticosteroids, hyperbaric oxygen (HBO) therapy, pentoxifylline and vitamin E therapy, and appropriate wound care. 3. Internal contamination - If a patient is externally contaminated, they have a higher risk of being internally contaminated. For internally contaminated patients, management should focus on decreasing absorption, enhancing elimination, and blocking distribution to target organs. - Treatment directed at internal contamination by particular radionuclides can include potassium iodide for radioactive iodine exposures, bicarbonate for uranium, Prussian blue for cesium and DTPA for plutonium and transuranics - See Table 138.5 with a list of the radionuclides of interest with their associated treatment options - Assess for Acute Radiation Syndrome Recap (follow in Figure 138.3) For more: - Canadian response links/plans http://nuclearsafety.gc.ca/eng/resources/emergency-management-andsafety/index.cfm - 24 hr CNSC duty officer telephone line: Phone 613-995-0479, the CNSC duty officer emergency telephone line, in the event of an emergency involving a nuclear facility or radioactive materials, including: - any accident involving a nuclear reactor, nuclear fuel facility, or radioactive materials - lost or damaged radioactive materials - any threat, theft, smuggling, vandalism or terrorist activity involving a nuclear facility or radioactive materials The CNSC Duty Officer emergency telephone line is available 24 hours a day, 7 days a week. BC Emergency Info: https://www2.gov.bc.ca/gov/content/safety/emergency-preparednessresponse-recovery

Describe the pre-hospital and ED management of frostbite

Priorities 1. Prevent re-freeze injury & thaw 2. Analgesia 3. Wound care 4. Tetanus prophylaxis 5. Consider if there is a role for thrombolytic therapy (IV or IA) 6. Post-thaw wound care and follow-up Prehospital: DO: - Remove from the cold environment - Prevent any thaw-refreeze cycles - Remove constricting and wet clothing - Insulate and immobilize the affected areas (unless you need to walk out on frozen feet) - If unable to evacuate thaw in 37-39 degree water DON'T - Use dry heat sources - Rub the tissue vigorously - Use heat forced air - Use fire ED management (Box 131.4) Prethaw Assess Doppler pulses and appearance. 1. Protect part—no friction massage. 2. Stabilize core temperature. 3. Address medical and surgical conditions. 4. Administer volume replacement as indicated. Thaw Provide parenteral opiate analgesia as needed. 1. Administer ibuprofen 400-600 mg (or aspirin, 325 mg). 2. Immerse part in circulating water at 37° C-39° C (98.6° F-102.2° F), monitored by thermometer. 3. Encourage gentle motion, but do not massage. Postthaw Dry and elevate. 1. Aspirate or débride clear vesicles. 2. Débride broken vesicles and apply topical antibiotic or sterile aloe vera ointment every 6 hours. 3. Leave hemorrhagic vesicles intact. 4. Administer tetanus prophylaxis if indicated. 5. Provide streptococcal prophylaxis if high risk. 6. Consider phenoxybenzamine in severe cases. 7. Perform imaging, including angiography, if thrombolysis may be indicated. 8. Carry out thrombolysis, if indicated and available. 9. Obtain admission photographs. A couple keys: - Water warmer than 39 degrees causes more pain, and no significant benefit -Thermal injury occurs if the water temperature is > 42 degrees - Usually 30 mins of immersion is needed - if in doubt keep it in longer (until distal erythema is noted) • Don't massage the tissue, but encourage gentle active movement • Have IV access available and note the risk for core temperature afterdrop and VF! - Elevate the affected extremity post thaw to prevent edema - Consider thrombolysis i. IV tPa or intra-arterial tPa (both with heparin) ii. Given within 24 hours of thawing (you have time!!) - Indications for thrombolysis: i. No contraindication to tPa ii. Risk for significant tissue loss (e.g. extending to proximal phalanx) - Other "unproven therapies" i. LMWH ii. Hyperbaric O2 iii. Iloprost (prostacyclin)

What is the best laboratory test for predicting the outcome following a radiation exposure?

Quantifying the absorbed dose of radiation can be challenging, especially in the emergency department (ED). Information on the radiation source, field strength, time of exposure, distance, shielding, and routes of exposure is often incomplete. Although radiation doses can be reconstructed at a later time by health physicists, emergency clinicians will most often rely on biodosimetry tools, such as time to vomiting and lymphocyte depletion kinetics. These online tools are available at www.remm.nlm.gov/ars_wbd.htm#vomit Best test at predicting outcome: - Lymphocyte depletion kinetics - ABSOLUTE LYMPHOCYTE COUNT - The 48-hour absolute lymphocyte count is the most important prognostic indicator and should be drawn on all suspected radiation exposure patients. Levels greater than 1200/µL indicate a clinically insignificant dose of radiation and an excellent prognosis. Levels less than 500/µL indicate a significant and possible lethal exposure. - A baseline complete blood count (CBC) with differential and absolute lymphocyte count should be obtained and repeated every 6 hours for the first 24 hours and at least daily thereafter. - The absolute lymphocyte count at 48 hours after exposure is a good predictor of radiation injury (Fig. 138.2). If the absolute lymphocyte count is greater than 1200 cells/µL, it is unlikely that the patient has received a clinically significant dose of radiation. If the absolute lymphocyte count falls between 100 and 500 cells/µL at 48 hours, a significant or even lethal dose of radiation should be suspected. A level in this range is an indication for neutropenic precautions. Weeks later, thrombocytopenia and anemia may develop because these cell lines are more radioresistant. BEST CLINICAL TEST = TIME TO ONSET OF VOMITING: - Be worried if after a radiation exposure the patient starts vomiting within the first 6 hours! (this suggests an exposure of at least 1.9 Gray Other tests: - Contamination survey instrument: - Geiger Muller detector: These detectors measure the presence of radioactive material in counts per minute. - Field strength devices - used to measure radiation fields at the event scene (done by radiation experts/nuclear physicist

What are the risk factors for hypothermia? List 6

SEE BOX 132.1 in Rosen's 9th Edition NOTE: Think about it as a problem of heat production or heat loss. Heat is mostly produced by cellular metabolism in the heart, liver, muscles and lost through the SKIN and LUNGS. The risk factors below relate to a body that isn't able to produce heat (elderly patient with liver disease, decreased cardiac output, thin skin on cardiac suppressing drugs) or something that puts someone at higher risk of exposure. Risk factors for hypothermia include: - Over exposure - Poor Health - Old Age - Drug Ingestion - Intoxicant Ingestion - Inadequate nutrition

What are five indications for active rewarming?

See Box 132.3 in Rosen's 9th Edition for the list of indications for active rewarming in the hypothermic patient Five Indications for Active Rewarming 1. Cardiovascular instability 2. Moderate to severe hypothermia (<32 degrees Celsius) 3. Inadequate rate of rewarming or failure to rewarm 4. Endocrine insufficiency 5. Traumatic or toxicological peripheral vasodilation 6. Secondary hypothermia impairing thermoregulation

Tips to avoid getting hit by lightning

See Box 134.5 in Rosen's 9th edition for tips to avoid lighting strikes - Squat!

List 4 types of electrical burns and 5 mechanisms of lightning injury

See Box 134.6 in Rosen's 9th edition for types of burns associated with electrical injury: - Entrance and exit site burns - Arc burns, kissing burns - Thermal burns - Flash burns Most electrical injuries result in skin burns, which fall into one or more of four patterns: (see table) - Usually the hand or wrist is affected - These burns are associated with much higher morbidity than similar appearing thermal burns ****great amount of damage underneath the surface of skin***** - Burns at entrance and exit sites will typically have a punctate appearance, with central depression and necrosis surrounded by a hyperemic border. Types: 1. Entry and exit burns - direct contact 2. Arc or "kissing burns" a. Kissing burns = when electricity jumps from skin surface to skin surface, typically across flexed areas of the body. Temperatures may reach 3500°C (6332°F) and cause severe damage. Arc burns are usually noted across the volar forearm and elbow and along the inner arm and axilla. 3. Thermal burns due to clothes catching on fire 4. Flash burns: skin burns caused by brief, intense flashes of light, electrical current, or thermal radiation. Shock goes off nearby. Cutaneous burns across the chest and upper abdomen hint at transthoracic current and a worse prognosis. Lightning injuries: - Direct/contact strike - Side flash / splash injury - Stride injury

List the ECG changes seen with lightning strikes (5).

See Box 134.7 in Rosen's 9th edition for electrocardiographic changes seen with lightning strikes. - ST elevation - QT interval prolongation - Atrial fibrillation - Inverted or flattened T waves - Myocardial infarction pattern without cardiac sequelae

List clinical findings associated with lightning exposure.

See Box 134.8 in Rosen's 9th edition for findings suggestive of a lightning strike: - Clothing wet from rain - Tears or disintegration of clothing - Multiple victims - Typical arborescent pattern of erythema or superficial linear or punctate burns - Tympanic membrane injury - Cataracts, especially in a younger patient - Magnetization of metallic objects on the body or clothing - Electrocardiographic changes Hx: - Multi-casualty incident at an outdoor event (usually circa thunderstorm) Px: - Wet clothing with tears in it - Lightning burn pattern on skin - Tympanic membrane injury - Cataracts Tests: - ECG changes - Magnetic properties of metal objects on the patient

What are key aspects of a diving history?

See Box 143-1 and Figure 143-8 - Dive profile - Depth and length of dive - **when did the symptoms first occur** - Ask members of the dive group Focus on treatment decisions, rather than exact diagnosis. Call your HB doc sooner!

List 5 potential injuries a diver can sustain in descent, at depth, and on ascent

See Figure 143-8 (8th) / 135.9 (9th) Descent: - head squeezed like a tube of toothpaste i. Middle Ear Barotrauma - Most common complaint among divers ii. Inner Ear Barotrauma iii. External Ear Barotrauma - Rare - due to wax in the auditory canal. iv. Facial barotrauma - Due to the dive mask eye-nose interface if the diver doesn't exhale through their nose. Can cause facial / conjunctival edema, petechial hemorrhages, subconjunctival hemorrhages. v. Sinus barotrauma - barosinusitis is the 2nd most common complaint among divers. Can occur on descent or ascent, causing facial pain/epistaxis At depth: - too much funny gas: huffing that helium balloon - Nitrogen narcosis - Oxygen toxicity - Contaminated gases - Hypothermia Ascent: - opening a shaken bottle of pop - Rapid up - Alternobaric vertigo i. Inability to equalize pressure in the middle ear during ascent, problematic with concurrent URTI ii. Increased pressure in the middle ear causes nystagmus Pulmonary over-pressurization syndrome - Air gas embolism - Pneumothorax - Pneumomediastinum - Pulmonary hemorrhage Barodontalgia - Air is trapped beneath a poorly filled dental cavity - and expands on ascent (benign and self limited) GI barotrauma - Rare problem. Suspect in someone with a diving history and abdominal pain - Due to expanding bowel gas due to carbonated beverages/large meals/gassy foods. Long and deep - DCS I or II - Arterial gas embolism

What are the prehospital management priorities for the hypothermic patient in each of these categories: mild, moderate and severe hypothermia?

Specific goals of prehospital care include prevention of further heat loss with insulation, avoidance of afterdrop, gentle handling, and transporting in a horizontal position. Patients should be actively rewarmed in the field, if possible. Mild: - Remove wet clothing, insulate with warm, dry clothes - Drink warm, sweet liquids - Encourage active movement - Hike out / transport to warm environment - Treat any other injuries (including cold-induced ones!) Moderate & Severe: - Text and figure below copied from: "Accidental Hypothermia" Douglas J.A. Brown, M.D., Hermann Brugger, M.D., Jeff Boyd, M.B., B.S., and Peter Paal, M.D. NEJM 2012. i. Careful handling of the patient, ii. Provision of basic or advanced life support, iii. Passive and active external rewarming, iv. Transport to an appropriate facility. - Detecting a pulse in a patient with hypothermia may be difficult, so signs of life and pulse should be checked carefully for 60 seconds. Persistent breathing or movement by the patient should prompt a strategy of watchful waiting, but if no signs of life are detected, then cardiopulmonary resuscitation (CPR) should be started. Full-body insulation and rewarming should be provided for all patients as long as it does not impede CPR or delay transport. For rewarming in the prehospital setting, only chemical, electrical, or forced-air heating packs or blankets provide a substantial amount of heat transfer (Table 3). Advanced airway management should be performed if indicated, since the risk of triggering a malignant arrhythmia is low. - Highly recommend checking out this figure as it covers all priorities: If you want some excellent summaries of Dr. Doug Brown's work regarding accidental hypothermia in the prehospital setting check out: http://drdougbrown.ca/

Describe the diving reflex

The diving reflex may also play a protective role in infant and child submersions. Activation of the diving reflex by fear or immersion of the face in cold water shunts blood centrally to the heart and brain. Apnea and bradycardia ensue, prolonging the duration of submersion tolerated without central nervous system (CNS) damage. The proposed protective effect of cold water immersion was unfortunately not seen in a study of 1094 drowning victims of all ages, where water temperature had no correlation.

What is the relationship between current, voltage and resistance? How does this relate to potential for injury from electrical and lightning injuries

The first part of this question relates to Ohm's law: Current is the flow of electrons down an electrical gradient. It is measured in units of ampere. According to Ohm's law, current is directly proportional to the voltage of the source and inversely proportional to the resistance of the material through which it flows. V = I R Hydraulic model But let's take a deeper dive and discuss the factors affecting electrical injury The degree of injury from electrical shock depends on multiple factors: "Voltages Can Cause ARC's" Voltage - Electrical potential difference between two points. See Joules law. - Joule's law, which describes the amount of thermal energy applied to tissues from electricity, is described by this formula: P I RT = 2 i. where I is the amperage, R is the resistance, and T is the duration (time) that the electricity is applied. As the formula indicates, voltage is not the only factor responsible for damage, but it is often the only property that is known in cases of electrical injury. As a result, injuries are conventionally classified as being caused by high- or low-voltage sources, with 1000 V as the dividing line. In the United States and Canada, household sources are low voltage, typically 120 or 240 V. Current pathway - Internally, current follows the path of least resistance, and the degree of burns seen on the surface typically underestimates the damage occurring below the surface. - Limb paths cause more local tissue damage. - Transthoracic pathways (arm to arm) are more likely to generate arrhythmias and have higher mortality rates than vertical currents (leg to arm) or straddle pathways (leg to leg). Contact duration - Tissue damage is directly proportional to duration of exposure regardless of voltage level. - Exposure times greater than the length of one cardiac cycle tend to generate arrhythmias, likely in a manner analogous to the R-on-T phenomenon. Amperage - This is the electrical current or flow of electrons down a voltage gradient (amount of water moving through the pipe per second). Higher current transfers more energy, aka heat into the person, causing more injury. Resistance of the tissue - Resistance is the degree to which a substance resists the flow of current; - Neurovascular tissue (high water content tissues) conduct well. Dry skin conducts very poorly. Current that is initially unable to pass through skin will create thermal energy and cause significant burns. Circuit type - AC vs. DC (AC is worst, more on this in question 3). Well there's a formula for these principles. - See Box 134.3 in Rosen's 9th edition. P = I^2RT I = V/R Current is the flow of electrons down an electrical gradient. It is measured in units of ampere. Resistance of body tissues (Box 134.4 in Rosen's 9th edition): Nerve (lowest) < Blood Vessels < Muscle < Skin < Tendon < Fat < Bone (highest)

List 6 factors contributing to the incidence of high-altitude illness

The incidence and severity of altitude illness are directly related To: 1. elevation 2. rapidity of ascent. Other variables influencing AMS development include: 3. prior acclimatization, 4. Individual genetic susceptibility, 5. sleeping elevation, 6. duration of stay

When post-dive is it ok to go on a trans-continental flight?

The post dive pre-flight surface interval = depends on the diver's repetitive group designator or residual nitrogen time. The time to flight interval depends on the number of dives, their depths and if no decompression limits were approached. You'd be safe if you didn't have any DCS or AGE symptoms if you waited 48 hrs....but as an example you'd be able to fly after 12 hrs of surfacing if you had less than 2 hrs of dive time in the last 48 hrs. This also brings up the questions about how to get a patient to decompressive therapy. Ideally the plane should be pressurized to < 1000 ft. A helicopter should fly <500 ft.

What is meant by wet bulb globe tempearture?

The wet bulb globe temperature heat index is an excellent meteorologic measure of environmental heat stress. It includes the effects of temperature, humidity, and radiant thermal energy from the sun. When climatic conditions exceed 25°C (77°F) wet bulb, even healthy people are at high risk during exercise. Above 28°C (82.4°F), exercise and strenuous work should be avoided or limited to extremely short periods. WBGT = 0.1 x Dry Bulb Temperature (DBT) + 0.7 x Wet Bulb Temperature (WBT) + 0.2 x Globe Temperature (GT) DBT represents the ambient air temperature, WBT the relative humidity, and GT the radiant heat. The equation for the WBGT reflects the critical importance of evaporative cooling for managing heat stress, as judged by the relative weight given to WBT. Measurements to determine the WBGT should be obtained about three to four feet off the ground on the playing field where the training session or sporting event will take place. Given the close association between WBGT and exertional heat illnesses, this measurement should be used to guide and modify the intensity and duration of exercise, the use of equipment (eg, football helmets and padding), the frequency of rest breaks, and hydration needs. Any person or group responsible for these kinds of decisions should establish an accurate method for determining WBGT on site and should not rely upon local weather stations or news reports. However, it is acceptable to use WBGT measurements performed regularly by experts within close proximity (approximately 10 miles or 16 km) of the site of athletic activity (eg, WBGT calculated daily by local airport meteorologists)." - Uptodate The heat strain index is widely accepted as an example of an index that includes environmental and physiologic factors. There are several variations and modified heat strain indices, with varying ease of use and accuracy. Cooling is best achieved by evaporation from the body surface; sweat that drips from the skin does not cool the body. Each liter of completely evaporated sweat dissipates 580 kcal of heat. The ability of the environment to evaporate sweat is termed atmospheric cooling power and varies primarily with humidity, but also with wind velocity. As humidity approaches 100%, evaporative heat loss ceases. - Rosen's 9th Edition, Chapter 133 (see Rosen's Box 133.1)

Describe the difference between MEBT, IEBT, ABV and Middle Ear DCS

These are all tricky entities - and the differential diagnosis of MEBT, IEBT, inner ear DCS, ABV includes all....See Table 135.2 (9th)

Differentiate between minor heat illness, heat exhaustion and heat stroke clinically

These things exist on a spectrum: in reality heat exhaustion and heat stroke can have overlapping symptoms. - when in doubt treat as a the most severe form! Minor Heat Illness: One of these presentations: - Heat rash - Heat edema - Heat syncope - Heat cramps - Normal temperature - No persistent CNS symptoms - Localised symptoms Heat Exhaustion: - Vague malaise, fatigue, headache - Core temperature often normal; if elevated, <40°C (104° F) - Mental function essentially intact; no coma or seizures - Tachycardia, orthostatic hypotension, clinical dehydration (may occur) - Other major illness ruled out - If in doubt, treat as heatstroke. - Vague CNS symptoms - Intact mental function - <40 deg. C. - Can proceed heat stroke Heat Stroke: - Exposure to heat stress, endogenous or exogenous - Signs of severe central nervous system dysfunction (coma, seizures, delirium) - Core temperature usually > 40.5°C (105° F), but may be lower - Hot skin common, and sweating may persist - Marked tachycardia - Sudden onset altered LOC - Hot skin +/- sweat - Major CNS dysfunction - Coma, seizures - Temp > 40.5 (CORE temp) - Multi-system tissue damage and organ dysfunction (major lab abnormalities) - Some say any temperature > 40 deg. C AND altered mental status or neurologic findings is sufficient to dx heat stroke Mild heat exhaustion and full-blown heat stroke represent extremes of the spectrum of heat illness, and intermediate cases may prove difficult to differentiate. Nevertheless, heat exhaustion should not be diagnosed in the presence of major CNS dysfunction* (eg, seizures, coma) or severe hyperthermia (40.5°C [104.9°F]). *Any type of dysfunction can occur: bizarre behaviour, opisthotonos, hallucinations, rigidity, cerebellar dysfunction*

List six types of heat-related illnesses

This is a review from this topic: Minor - Heat rash - Heat cramps - Heat syncope - Heat edema Major - Heat exhaustion - Heat stroke i. Exertional vs. Classic *classic heatstroke include predisposing factors or medication, older population, sedentary lifestyle, anhidrosis, normoglycemia, mild coagulopathy, mild elevation in creatine kinase level, oliguria, mild acidosis, and occurrence during heat waves. Diaphoresis, hypoglycemia, disseminated intravascular coagulation, and marked lactic acidosis are characteristics of exertional heatstroke." - Rosen's 9th Edition, Chapter 133

List 3 routes of exposure to radiation

This is spaced repetition from question 1; but Rosen's describes three main processes: 1. Irradiation (e.g. external exposure of a person or object to a radioactive source) - An object doesn't become radioactive unless neutron activation occurs. So, when a person is irradiated, such as a patient who has just received a CT scan or x-ray, no hazard exists to medical personnel who come into contact with the patient. 2. Contamination - exposure to radioactive particulate matter (alpha and beta particles). - Contamination usually occurs externally but may be internal if the radioactive material is inhaled and deposited in the lungs. - Contamination is not an acute threat to the life of the patient or the provider, and its presence should not preclude institution of lifesaving measures. The radioactive particulate matter may emit radiation with an effect that is directly related to the time of exposure, distance from the source, and type of contamination. 3. Incorporation - when a radioactive material is taken up by a tissue, cell, or organ. This can occur through ingestion, inhalation, or absorption via an open wound. - As in radioiodine therapy for an overactive thyroid or inhalation of radioactive material after bomb blast or reactor explosion

How do you treat HAPE?

This is the #1 fatal form of high altitude illness. See Box 136.4 in Rosens 9th - Usually occurs at 2500-3000m (but can occur at lower elevations) - Investigations usually helpful to work-up other causes of acute dyspnea at altitude (u/s, CXR, ECG) Viagra: consider as prophylaxis in those who have had HAPE before: 50mg q8h Treatment: 1. Stop ascending, rest, keep warm 2. Oxygen (or hyperbaric/normobaric oxygen via gamow) 3. Descent (usually at least ~1000m, but as much as practically possible) 4. Nifedipine a. Unlike pulmonary edema secondary to acute CHF, HAPE does not result from excessive intravascular volume or failed cardiac pump function. As such, diuretic therapy has no role in the treatment of HAPE and may further exacerbate volume loss in patients who are already intravascularly depleted. b. lowers pulmonary artery pressure, pulmonary blood volume, and pulmonary vascular resistance or enhance alveolar fluid clearance c. Works well for prophylaxis and treatment d. **good therapy to have on hand if descent is impossible or no oxygen is available** e. Treatment with 30 mg of a slow-release nifedipine preparation administered twice daily is effective. Patients should be monitored for the development of hypotension during nifedipine administration. 5. Other medications a. Unstudied for acute treatment, may be helpful for prevention i. phosphodiesterase type 5 inhibitors (including tadalafil and sildenafil) ii. Beta-adrenergic agonists to help with alveolar fluid clearance (salmeterol 125 μg inhaled twice daily) The mainstay of HAPE treatment remains immediate oxygen (if it is available) and descent. Should these treatments not be available, nifedipine should be initiated. No compelling evidence suggests the concurrent use of these medications with oxygen has additional benefit beyond the use of oxygen alone. Of note, the radiographic findings of cardiomegaly, bat-wing distribution of infiltrates, and Kerley B lines, which are typical of cardiogenic pulmonary edema, are absent in cases of HAPE. Patients may be able to re-ascend (generally in 2 to 3 days) when symptoms resolve and oxygen levels remain acceptable off supplemental oxygen at rest and with mild exercise. Re-ascent with pulmonary vasodilator medication may be considered. Other prevention therapies: - PDE5 inhibitors - Dexamethasone - Salmeterol - Acetazolamide (to help with acclimatization, prevent HAPE, and reduce pulmonary vasoconstriction)

How are perioral electrical burns managed? List three early and three late complications.

Toddlers and young children sustain orofacial injuries after chewing or sucking on electrical cords or from lingual contact with sockets. Full-thickness burns may be sustained on the mucous membranes and lips, with destruction to the tongue and teeth as well. Injuries to the oral commissure produce cosmetic difficulties and, more significantly, the well-recognized complication of delayed labial artery bleeding, typically occurring 2 days after injury, when the resultant eschar separates from the wound. Early: - Thermal burn with soft tissue swelling - Dehydration - Airway compromise Late: - Perioral scarring - Dental loss - AVN of the bone - DELAYED labial artery bleeding

What is an Arterial Gas Embolism (AGE)

Type of "decompression related illness" - SERIOUS form of DCS - Air bubbles forced across alveolar-capillary membrane → pulmonary Vein → LA-LV then rest of the body ******most common cause of death in divers******** - Especially with cerebral or coronary embolisation - Can occur even after a short, shallow dive. Symptoms: -Sudden onset - ***cerebral symptoms*** - Loss of consciousness - h/a, confusion, convulsions, motor/sensory loss, alteration of normal consciousness, seizures, visual changes, ataxia - Cardiac arrhythmias / arrest The 10 minute rule of thumb "Any diver breathing compressed air at any depth underwater who surfaces unconscious or who loses consciousness within 10 minutes of reaching the surface should be assumed to be suffering from AGE." DCS typically presents after 10 minutes and may present up to 24 hrs

Describe the skin injuries associated with lightning and electricity

We discussed the skin findings associated with electrical injury in question 4: Types: 1. Entry and exit burns - direct contact 2. Arc or "kissing burns" a. Kissing burns = when electricity jumps from skin surface to skin surface, typically across flexed areas of the body. Temperatures may reach 3500°C (6332°F) and cause severe damage. Arc burns are usually noted across the volar forearm and elbow and along the inner arm and axilla. 3. Thermal burns due to clothes catching on fire 4. Flash burns: skin burns caused by brief, intense flashes of light, electrical current, or thermal radiation. Cutaneous burns across the chest and upper abdomen hint at transthoracic current and a worse prognosis. When it comes to lightning - less than 5% are deep skin burns and instead most skin findings are from flashover effects. current preferentially flows over the integument rather than through it (following the path of least resistance). The result is the arborescent or fern like patterns of erythematous streaks (typically first-degree burns) that have been termed Lichtenberg figures See Fig. 134.4 in Rosen's 9th edition. Deeper burns may occur at the direct point of contact or wherever metal is involved (due to superheating), such as with a belt buckle or jewelry. Clothing may catch on fire, resulting in thermal burns. Unlike conventional high-voltage electrical exposures, exit wounds are not seen, and the overall effects are much less severe.

List 6 risk factors of decompression sickness. List the 2 types of decompression sickness (clinical features)

What is the pathophysiology of decompression sickness? A great explanation from Uptodate: - As a diver descends and breathes air under increased pressure, the tissues become loaded with increased quantities of oxygen and nitrogen as predicted by Henry's law. - As the diver returns to the surface, the sum of the gas tensions in the tissue may exceed the ambient pressure and lead to the liberation of free gas from the tissues in the form of bubbles; the location of bubble formation is somewhat dependent upon tissue characteristics. The liberated gas bubbles can alter organ function by blocking vessels, rupturing or compressing tissue, or activating clotting and inflammatory cascades. The volume and location of these bubbles determine if symptoms occur. I think about it like using a SodaStream...you charge the water with pressure and carbon dioxide, and then when you take the cap off and start drinking the CO2 comes out of solution. DCS = decompression sickness - Incidence: 2.8 / 10000 (a few recent deaths!) Risk factors: - Patient: i. Obesity ii. Older age iii. Fatigue, dehydration, fever iv. Heavy exertion v. Male gender vi. ETOH / Tobacco use vii. Presence of a Patent Foramen Ovale - Most sport diver's aren't screened for this - and some may even have normal bubble-ECHO studies: suggesting that the PFO may only open at high ambient pressures - Energy: Repetitive dives within several hours of each other - Environment: i. Longer dive (>time) ii. Deeper dive - Almost never occurs with dives < 6 meters deep iii.Cold ambient temperature iv. High altitude diving v. Flying after diving The US Navy has developed dive tables - which are usually programed into dive "computers" - that set limits to prevent DCS. If these limits are exceeded a diver must do "decompression stops underwater" to off-gas nitrogen built up in the tissues. But these are not a guarantee that DCS will not occur. Divers can develop DCS even when they are within any calculated no-decompression limit. Presentation of DCS: - Usually manifests within hours of surfacing - 60% occur within 3 hrs, and almost all occur within 24 hr. This can be hastened by ascending to altitude or flying - and sometimes can present DAYS after diving. - Symptoms [ Fig 143-7 ] - in order of occurrence i. Pain (joint, muscle, girdle) ii. Numbness, paresthesias iii. Constitutional: h/a, lightheadedness, inappropriate fatigue, malaise, N/V iv. Dizziness/vertigo v. Motor weakness vi. Skin findings vii. Muscular stiffness, pressure, cramps viii. Mental status changes ix. dyspnea/cough x. Auditory xi. Bladder bowel "Decompression-related illness" - all get treated the same!!

Describe the clinical differentiation between frostnip, frostbite, trenchfoot, and perino:

[1] Frostnip - Superficial freezing injury = transient numbness and tingling that resolves after rewarming - NO tissue destruction [2] Frostbite - This is the big one for this episode! Frozen tissue! - #1 presenting symptom = numbness = loss of pain, temperature and light touch sensation - Any of the commonly exposed areas - Also: i. Clumsiness ii. "Block of wood" sensation - The initial presentation of frostbite is usually deceptively benign - (Assuming the person doesn't have an obviously frozen hand) - Violaceous, waxy, white, pale yellow i. Unable to move the skin over bony tissues - Rapid warming usually causes hyperemia - even in severe cases of frostbite Post rewarming: - Good prognosis = i. Normal sensation, warmth and colour - Concerning prognosis: i. Bleb formation ii. Residual violaceous hue iii. Hemorrhagic vesicles iv. Lack of edema formation v. Eschar and mummification There are at least three different tissue classification models - Superficial vs deep injury (based on whether tissue is lost) - 1st through 4th degree (not recommended because it is inaccurate and may mislead management) - Grade 1-4: based on response to rapid rewarming (see below) From uptodate: A more useful clinical prediction tool has been developed for frostbite of the hands and feet, based on the level at which skin lesions are noted after rapid rewarming in warm water. The categories are as follow: i. Grade 1 frostbite is characterized by no cyanosis on the extremity. This predicts no amputation and no sequelae. ii. Grade 2 involves cyanosis isolated to the distal phalanx. This predicts only soft tissue amputation and fingernail or toenail sequelae. iii. Grade 3 frostbite is characterized by intermediate and proximal phalangeal cyanosis. This predicts bone amputation of the digit and functional sequelae. iv. Grade 4 frostbite involves cyanosis over the carpal or tarsal bones. This predicts bone amputation of the limb with functional sequelae. PMID: https://www.ncbi.nlm.nih.gov/pubmed?term=11769921 [3] Trenchfoot (immersion injury) - Injury occurs due to immersion or damp conditions over days (wet socks on a long hike for days or weeks) - Neurovascular damage, blistering and tissue loss can occur Stages: 1. Cold exposure - numbness - Red to pale to white tissue - Lasts until out of the cold 2. Rewarming - mottling, pale blue - Cold and numb and progresses to pain and edema - Can last days 3. Hyperemia: - Hot, red and prolonged cap refill - Vasomotor paralysis - Severe pain, hyperalgesia - Edema and bullae formation - Can last weeks to months 4. Post-hyperemia - Normal appearance unless tissue lost - May have chronic pain [4] Chilblains (Perino) - Due to repetitive exposure to cold conditions or in someone with underlying disease - Look like cold sores that appear within 24 hrs after exposure to cold i. Face, hands, feet, tibia - Risk groups: Young women, Raynaud's / SLE / APLAb pts. - Symptoms: burning, pruritus, erythema, edema. - Resolves in 1-2 weeks. - Analgesia; consider nifedipine

Describe the types of major heat illness

a) Heat exhaustion - A clinical syndrome of volume depletion during heat stress. Usually individuals working in a hot environment who don't drink enough water and electrolytes to replace the losses - Two main types (but usually it is a mixed type) : i. Salt depletion - relative salt deficit due to excessive free water intake (slower to develop). It differs from heat cramps in that systemic symptoms occur. Symptoms are similar to those seen in water depletion heat exhaustion; the body temperature usually remains nearly normal. ii. Water depletion type - loss of primarily free water - Sx: i. Vague. ii. weakness, fatigue, frontal headache, impaired judgment, vertigo, nausea and vomiting and, occasionally, muscle cramps. Orthostatic dizziness and syncope can occur. Sweating persists and may be profuse. The core temperature is only moderately elevated, usually below 40°C (104°F), and signs of severe CNS dysfunction (eg, altered mental status) are not present. - Heat exhaustion: diagnosis (Box 133.3) i. Vague malaise, fatigue, headache ii. Core temperature often normal; if elevated, <40C iii. Mental function essentially intact; no coma or seizures iv. Tachycardia, orthostatic hypotension, clinical dehydration (may occur) v. Other major illnesses ruled out vi. If in doubt, treat as heatstroke - Labs: i. HypoNa ii. HypoCl iii. CPK elevation Heat exhaustion: management (Box 133.4) i. Rest ii. Cool environment iii. Assessment of volume status - orthostatic changes, BUN level, Hct, serum sodium concentration iv. Fluid replacement - normal saline to replete volume if patient is orthostatic; replace free water deficits slowly to avoid cerebral oedema v. Healthy young patients are usually treated as outpatients, consider admission if the patient is older, has significant electrolyte abnormalities, or would be at high risk for recurrence if discharged b) Heat stroke - This is a CATASTROPHIC life-threatening emergency when the body's thermoregulatory systems fail. - Neurologic dysfunction is a hallmark of heat stroke, and cerebral edema is common. i. Failure of compensatory peripheral vasodilation and central vasoconstriction leads to cerebral ischemia - The cardiovascular system is also stressed leading to central vasoconstriction and peripheral vasodilation. i. compensatory vasoconstriction of the splanchnic and renal vasculatures. The resulting splanchnic and renal ischemia may explain the nausea, vomiting, and diarrhea observed in runners after a marathon. Hepatic damage is a consistent feature of heatstroke, and its absence should cast doubt on the diagnosis. - Hematological system dysfunction: i. Abnormal hemostasis is manifested clinically by purpura, conjunctival hemorrhage, melena, bloody diarrhea, hemoptysis, hematuria, myocardial bleeding, or hemorrhage into the CNS. Diarrhea, probably caused by intense splanchnic vasoconstriction, is commonly seen. Cooling aggravates the diarrhea, creating an unpleasant treatment problem. Pancreatitis is described, with elevated serum amylase and lipase levels. - Heatstroke: diagnosis (Box 133.5) i. Exposure to heat stress, endogenous or exogenous ii. Signs of severe CNS dysfunction (coma, seizures, delirium) iii. Core temperature usually >40.5C, but may be lower iv. Hot skin common, and sweating may persist v. Marked elevation of hepatic transaminase levels - See Rosen's Figure 133.5 for pathogenesis of haemmorhage

List the four types of minor heat illness - include their clinical features and management

a) Prickly heat - aka: Miliaria rubra, lichen tropicus, heat rash - Acute inflammatory skin disorder: the blockage of sweat gland pores by macerated stratum corneum and secondary staphylococcal infection. - intensely pruritic vesicles on an erythematous base. The rash is confined to clothed areas. Rash can persist for weeks - Chlorhexidine in a light cream or lotion is the antibacterial treatment of choice during the acute phase. Salicylic acid, 1% tid, can be applied to localized affected areas to assist in desquamation, - Prickly heat can be prevented by wearing light, loose fitting, clean clothing and avoiding situations that produce continuous sweating. The routine use of talcum or baby powder should be avoided. b) Heat cramps - Not the same thing as exercise associated muscle cramps - Heat cramps are brief, intermittent, and often severe muscle cramps occurring typically in muscles that are fatigued by heavy work. Heat cramps appear to be related to a salt deficiency. - They usually occur during the first days of work in a hot environment and develop in persons who produce large amounts of thermal sweat and subsequently drink copious amounts of hypotonic fluid. - Clinical Features: athletes, roofers, steelworkers, coal miners, field workers, and boiler operators are among the most common victims of heat cramps. Heat cramps tend to occur after exercise, when the victim stops working and is relaxing - Usually have total body hyponatremia and hypochloremia. - Most cases respond well to PO salt intake (tablets dissolved in oral solution). IV NS may be needed in severe cases. - Essentials of diagnosis (Box 133.2) i. Cramps of most worked muscles ii. Usually occur after exertion iii. Copious sweating during exertion iv. Copious hypotonic fluid replacement during exertion v. Hyperventilation not present in cool environment c) Heat oedema - hydrostatic pressure and vasodilation of cutaneous vessels, combined with some degree of orthostatic pooling, lead to vascular leak and accumulation of interstitial fluid in the lower extremities. i. Increase aldosterone levels further encourage fluid retention - Swollen feet/ankles i. Usually in adults with existing chronic disease - Important DDx in this population: CHF, liver disease, infections, or DVT's - Managed expectantly. i. Usually the edema resolves with acclimatization ii. No role for diuretics iii. Support stockings and leg elevation may help d) Heat syncope - In warm environments an increased volume of blood is in the peripheral circulation - and decreases central circulation i. This is worsened by long periods of standing → placing people at risk for inadequate central venous blood return and a decrease in cerebral perfusion leading to a LOC - Usually a multifactorial disorder, but elderly people are more fragile and at risk for it i. Do that full hx, physical and let that guide your workup - True heat syncope is self-limited and can be prevented with acclimatization, active muscle movement, compression garments, and a euvolemic state

List 10 predisposing factors for frostbite

a. All you outdoor adventurers! b. Homeless or displaced persons c. Military or service people in the outdoors d. Any Canadian, Alaskan or northern American! See Box 131.2 in Rosen's for a Comprehensive List Physiologic, Mechanical, Environmental, and Psychological Factors Physiologic 1. Genetic 2. Core temperature 3. Previous cold injury 4. Acclimatization 5. Dehydration 6. Overexertion 7. Trauma—multisystem, extremity 8. Dermatologic disease 9. Physical conditioning 10. Diaphoresis, hyperhidrosis 11. Hypoxia Mechanical 1. Constricting or wet clothing 2. Tight boots 3. Vapor barrier, Aveolite liners 4. Inadequate insulation 5. Immobility or cramped positioning Psychological 1. Mental status 2. Fear, panic 3. Attitude 4. Peer pressure 5. Fatigue 6. Intense concentration on tasks 7. Hunger, malnutrition 8. Intoxicants Environmental 1. Ambient temperature 2. Humidity 3. Duration of exposure 4. Wind chill factor 5. Altitude and associated conditions 6. Quantity of exposed surface area 7. Heat loss—conductive, evaporative 8. Aerosol propellants 9. Cardiovascular 10. Hypotension 11. Atherosclerosis 12. Arteritis 13. Raynaud's syndrome 14. Cold-induced vasodilation 15. Anemia 16. Sickle cell disease 17. Diabetes 18. Vasoconstrictors, vasodilators


संबंधित स्टडी सेट्स

LearningCurve 9b) Infants and Children

View Set

Mastering A & P Chapter 10- Questions

View Set

The Bits and Bytes of Computer Networking. Week 6: Troubleshooting and the Future of Networking

View Set

Chapter 32: Health Assessment of Children

View Set

History- Fall of Vietnam, Fall Of Vietnam

View Set

Sadlier Vocabulary Workshop Level C: Unit 5; Completing the Sentence

View Set

WGU : introduction to IT : CHPT 5 computer software

View Set